Biologia Spellbook Flashcards

1
Q

In eukaryotes, the ultimate yield of ATP from NADH is lower when the NADH is produced by:

A) Glycolysis

B) Pyruvate Dehydrogenase Complex

C) Krebs Cycle

D) Electron Transport and Oxidative Phosphorylation

A

(A) because in order for the NADH produced during glycolysis to be utilized by the ETC in ATP formation, the electrons must be shuttled into the mitochondria. When shuttled in, the electrons are typically used to reduce Coenzyme Q and bypass NADH dehydrogenase. This results in pumping of fewer protons out of the mitochondrial matrix, and ultimately in fewer ATP being formed.

How well did you know this?
1
Not at all
2
3
4
5
Perfectly
2
Q

Some inhibitors bind irreversibly to enzymes by covalent attachment. Would the kinetics seen under these conditions (V vs [S] curve) be similar to those seen with a reversible noncompetitive inhibitor?

A) Yes, because it would reduce the Km of the reaction

B) Yes, because the net effect would be a loss of active enzyme available for the reaction

C) No, because if enough substrate binds to the active site the reaction will reach Vmax

D) No, because Km will increase and Vmax will stay the same, similar to competitive inhibition

A

(B) An irreversible inhibitor will permanently deactivate the enzyme, reuducing effective enzyme concentrations.

How well did you know this?
1
Not at all
2
3
4
5
Perfectly
3
Q

Some enzymes can modify their substrate and by this means, regulate its activity. In many instances, these modifications are not permanent since other enzymes can reverse them. Which of the following categories of enzymes will irreversibly modify their substrate?

A) A kinase

B) A protease

C) A phosphatase

D) An acetylase

A

(B) Proteases are enzymes that cleave their substrates at specific sites, permeantnly removing part of the protein. A kinase adds a phosphate to its substrate, which can be removed by a phosphatase and an acetylase will add an acetyl group which can be removed.

How well did you know this?
1
Not at all
2
3
4
5
Perfectly
4
Q

A common cause of strep throat is a Streptococcal pharyngitis infection. When S. pharyngitis undergoes aerobic cellular respiration, which of the following regions would have the lowest pH?

A) Matrix of the mitochondria

B) Intermembrane space of the mitochondria

C) Cytoplasm

D) Extracellular

A

(D) Prokaryotes do not have any membrane bound organelles such as mitochondria, so the protons return to the cytoplasm through the ATP synthase on the plasma membrane, thereby producing ATP. Thus, the most acidic place is outside the cell.

How well did you know this?
1
Not at all
2
3
4
5
Perfectly
5
Q

Salicyclic acid (aspirin) if taken in excess may act as an uncoupling agent. Uncoupling agents increase the permeability of the inner mitochondrial membrane, resulting in the dissipitation of the proton gradient. Which of the following would most likely be true in the presence of an uncoupling agent?

A) Electron transport at the inner mitochondrial membrane would cease

B) The energy from the proton-motive force would likely be dissipitated as heat rather than in producing ATP from ADP

C) H+ ions would flow through the inner membrane into the intermembrane space

D) There would be an increase in biosynthesis

A

(B) The proton gradient obtained by the electron transport chain is necessary in order to create ATP by the ATP synthase. This unharnessed energy would be dissipirated into heat, and since the ions would pass through the ATP synthase, less ATP would be created.

How well did you know this?
1
Not at all
2
3
4
5
Perfectly
6
Q

For a given enzyme concentration at a low substrate concentration, how does reaction rate change as the substrate concentration increases?

A) Logarithmically

B) Linearly

C) Exponentially

D) Indirectly

A

(B) At low substrate concentrations, the reaction rate increases linearly as the substrate concentration increases. Near saturation, the reaction rate begins to level off and does not change regardless of how much substrate it added.

How well did you know this?
1
Not at all
2
3
4
5
Perfectly
7
Q

A new enzyme inhibitor for Enzyme X has been discovered. Keeping the enzyme concentration constant, experiments show that the addition of large amounts of substrate causes the enzyme-substrate-inhibitor reaction to occur at the same maximum reaction rate as an enzyme substrate reaction. This is because:

A) The inhibitor binds at the active site and the addition of large amounts of substrate out-competed the enzyme inhibitor via noncompetitive inhibition

B) The inhibitor binds at an allosteric site and the addition of large amounts of substrate out-competed the enzyme inhibitor via noncompetitive inhibtion

C) The inhibitor binds at the active site and the addition of large amounts of substrate out-competed the enzyme inhibitor via competitive inhibition

D) The inhibitor binds at an allosteric site and the addition of large amounts of substrate out-competed the enzyme inhibitor via competitive inhibition

A

(C) The question states that large amounts of substrate added can out-compete the inhibitor and result in the same Vmax which is characteristic of competitive inhibition. Then, the inhibitor only binds at an active site. Noncompetitive inhibition occurs when an enzyme inhibitor binds to an allosteric site.

How well did you know this?
1
Not at all
2
3
4
5
Perfectly
8
Q

Which of the following is NOT an example of feedback inhibition?

A) The effect of estrogen on luteinizing hormone one week before ovulation

B) The effect of high serum calcium on parathyroid hormones

C) The effect of estrogen on luteinizing hormone one day before ovulation

D) The effect of cortisol on ACTH

A

(C) During the early stages of the menstrual/ovulatory cycle, lower levels of estrogen inhibit the release of luteinizing hormone. However, as ovulation approaches, high levels of estrogen stimulate the release of luteinizing hormone. High serum calcium inhibits parathyroid hormones and cortisol inhibits ACTH.

How well did you know this?
1
Not at all
2
3
4
5
Perfectly
9
Q

Carbon dioxide is an allosteric inhibitor of the formation of oxyhemoglobin. Which of the following best describes its effects on the binding of oxygen to hemoglobin?

A) It decreases the affinity of hemoglobin for oxygen and shifts the oxygen dissociation curve to the left

B) It decreases the affinity of hemoglobin for oxygen and shifts the oxygen dissociation curve to the right

C) It increases the affinity of hemoglobin for oxygen and shifts the oxygen dissociation curve to the left

D) It increases the affinity of hemoglobin for oxygen and shifts the oxygen dissociation curve to the right

A

(B) An inhibitor of the formation of oxyhemoglobin would have to decrease the affinity of hemoglobin for oxygen and lowered affinity means that the hemoglobin will be less saturated so a right-shifted curve is correct.

How well did you know this?
1
Not at all
2
3
4
5
Perfectly
10
Q

Protein phosphatases regulate the activity of metabolic pathways by:

A) proteolytic cleavage

B) allosteric regulation

C) constitutive activity

D) covalent modification

A

(D) A protein phosphatase removes a phosphate from a protein; this is a covalent modification.

How well did you know this?
1
Not at all
2
3
4
5
Perfectly
11
Q

Carbon monoxide is a competitive inhibitor of the formation of oxyhemoglobin. Which of the following best describes its effects on the binding of oxygen with hemoglobin?

A) It binds to the active site and it decreases Vmax

B) It binds to an allosteric site and it decreases Vmax

C) It binds to the active site and it has no effect on Vmax

D) It binds to an allosteric site and it has no effect on Vmax

A

(C) Competitive inhibitors bind to the active site of enzymes. Competitive inhibitors reduce V, but do not affect Vmax.

How well did you know this?
1
Not at all
2
3
4
5
Perfectly
12
Q

A competitive inhibitor of eukaryotic RNA polymerase III would have the greatest effect on:

A) replication

B) reverse transcription

C) translation

D) mutation

A

(C) RNA polymerase II transcribes tRNA which then carries amino acids to ribosomes for use in translation . Thus, polymerase plays no role in replication and reverse transcription. Blocking the action of this enzyme would not alter the base sequence, so mutation would not be affected.

How well did you know this?
1
Not at all
2
3
4
5
Perfectly
13
Q

In the lac operon, transcription is regulated by a repressor protein and only takes place in the presence of lactose. Which of the following statements is correct?

A) The repressor protein binds to the promoter site to inhibit transcription

B) Lactose binds to the promoter site to initiate transcription

C) Lactose binds to the repressor protein to inhibit transcription

D) The repressor protein binds the operator site to inhibit transcription

A

(D) The lac operon includes an operator site to which the repressor protein binds. The operator protein is located between the promoter region and the start transcription site. When the repressor is bound, RNA polymerase cannot move forward to the start site, thus transcription is inhibited. Lactose binds to the repressor protein at an allosteric site, causing a conformational change so that the repressor protein can no longer bind to the operator so RNA polymerase can move forward to the start site and transcription will occur.

How well did you know this?
1
Not at all
2
3
4
5
Perfectly
14
Q

Which of the following could not be caused by a single point mutation in the DNA?

A) Ala-Gln-Cys-Asp-Leu -> Ala-Gln

B) Ala-Gln-Cys-Asp-Leu -> Ala-Gln-Cys-Asp-Leu

C) Ala-Gln-Cys-Asp-Leu -> Ala-Gln-Cys-His-Lys

D) Ala-Gln-Cys-Asp-Leu -> Ala-Gln-Cys-His-Leu

A

(C) A point mutation is a single base pair substation. There are few possibilities that can result if a single base is substituted. If the new codon is now a STOP codon, the polypeptide will be truncated—which is a point mutation. If the new codon codes for the same amino acid before the mutation, then the silent mutation has occurred and no change will be seen (Choice B). If the mutation leads to a single new amino acid, then a missense point mutation has occurred (Choice D). There was a frameshift mutation, which can come from a single point mutation.

How well did you know this?
1
Not at all
2
3
4
5
Perfectly
15
Q

Which DNA base pair requires the most energy to break?

A) A-T

B) C-A

C) G-C

D) U-A

A

(C) Guanine and cytosine base pairs involve three hydrogen bonds whereas adenine and thymine only involves two hydrogen bonds.

How well did you know this?
1
Not at all
2
3
4
5
Perfectly
16
Q

Which of the following is NOT a similarity between replication and transcription

A) Both processes occur with the same fidelity

B) Polymerization in both processes is based on reading a template

C) A pyrophosphate is removed from every nucleotide as polymerization occurs

D) Both processes occur in the 5’ to the 3’ direction

A

(A) Because RNA polymerases do not proofread, transcription is less accurate. DNA polymerase moves along in the 3’ to 5’ direction while the new chain is made in the 5’ to the 3’ direction.

How well did you know this?
1
Not at all
2
3
4
5
Perfectly
17
Q

Telomeres are guanine-rich caps on the ends of each chromosome. Which of the following is the most likely function of a telomere?

A) High guanine content stabilizes parental strands to prevent excess tension during DNA unwinding

B) Protect the ends of the chromosomes from damage due to incomplete replication

C) Provide a site for helicase attachment

D) Seal the gaps left by Okazaki fragments in the lagging strand

A

(B) Because DNA polymerase can only elongate DNA from a primer, the ends of the lagging strands do not replicated. Thus, with each round of replication, the chromosomes get shorter. Once the telomere are shortened, the cells enter senesces (hibernation) and are marked for destruction.

How well did you know this?
1
Not at all
2
3
4
5
Perfectly
18
Q

Which of the following has 3’ to 5’ exonuclease activity but NOT 5’ to 3’ exonuclease activity

A) DNA pol III

B) DNA pol I

C) RNA pol II

D) RNA pol III

A

(A) RNA polymerases do not have exonuclease activity because they do not proofread, and DNA polymerase I has both exonuclease activities.

How well did you know this?
1
Not at all
2
3
4
5
Perfectly
19
Q

Which of the following physiological responses would be expected to occur in a patient with a severe peanut allergy?

I. Vasodilation

II. Reddening of the skin and increased skin temperature

III. Increased glomerular filtration

A

I and II. Vasodilation is where the vessels constrict in order to increase the temperature of the body as the blood cannot flow to cool off.

How well did you know this?
1
Not at all
2
3
4
5
Perfectly
20
Q

A person with a known allergy to Brazil nuts inadvertently ingests some of these nuts. Which of the following would be expected within 3 hours of the ingestion?

I. Levels of epinephrine would increase in the blood

II. Levels of glucose would increase in the blood

III. Levels of glucagon would decrease in the blood.

A

II and III. Ingestion of carbohydrates increases glucose levels in the blood and decreases glucagon levels in the blood.

How well did you know this?
1
Not at all
2
3
4
5
Perfectly
21
Q

Ribozymes and Enzymes differ principally in what way?

A

Ribozymes are built from nucleic acids while enzymes are built from amino acids.

How well did you know this?
1
Not at all
2
3
4
5
Perfectly
22
Q

Does this part of the passage refer to the innate or adaptive immune system?

“This is the first layer that a pathogen will encounter in the human body. This system consists of macrophages, mast cells, leukocytes, and other cell types. These cells provide an imediate, but generally nonspecific response to a pathogen. As such, there is no immunological memory of a pathogen the body was previously exposed to.”

A

Innate immune system.

How well did you know this?
1
Not at all
2
3
4
5
Perfectly
23
Q

Does this part of the passage refer to the adaptive or innate immune system?

“This system provides a more specific response to the pathogen. Due to the necessary alteration in gene expression, this immune response results in a time delay between pathogen exposure and maximal immune response. This immune system provides an immunological memory for pathogens to which the body was previously exposed. This immunological memory has been hypothesized to be a cause of sensitization in exposure to allogens.”

A

Adaptive immune system.

How well did you know this?
1
Not at all
2
3
4
5
Perfectly
24
Q

In which of the following individuals would the removal of the thymus have the most deleterious effect on the individual’s immune system?

A) A 5 year old child

B) A 35 year old man

C) A 35 year old woman

D) A 55 year old woman

A

(A) The thymus is critical for development of the immune system in children and adolescents. The thymus atrophies after adolescence. Thus, removal of the thymus in a 5 year old child will affect that child’s immune system the most.

How well did you know this?
1
Not at all
2
3
4
5
Perfectly
25
Q

Although rare, exposure of the fetus to teratogens results in compromised neuronal development and may lead to premature neuron death. This premature death often triggers paralysis of muscles. In which layer of the embryo must the teratogen act for this to occur?

A

Ectoderm, where the nervous system development occurs.

How well did you know this?
1
Not at all
2
3
4
5
Perfectly
26
Q

Surgical treatment to move the ulnar nerve to reduce pinching must involve cutting which structure?

A) epiphysis of the ulna bone

B) smooth muscle of the forearm

C) tendon and smooth muscle of the forearm

D) fibrous connective tissue

A

(D). To reach the ulnar nerve, fibrous connective tissue must be cut. Fibrous connective tissue holds the ulnar nerve in place. Epiphysis of bones (growth plate) would not be cut. Smooth muscle would only be cut if an artery needed to be cut.

How well did you know this?
1
Not at all
2
3
4
5
Perfectly
27
Q

Conn’s syndrome is characterized by excessive levels of aldosterone, which normally stimulates Na+ reabsorption from the proximal convoluted tubules of the kidneys. How would blood volume and pressure change in Conn’s syndrome?

A

Excessive aldosterone levels will cause high Na+ levels which will increase blod volume and pressure by its osmotic effects.

How well did you know this?
1
Not at all
2
3
4
5
Perfectly
28
Q

Cellular repolarization in neurons is most correctly described by which of the following statements?

A) Na+ ions move out of the cell

B) Na+ ions move into the cell

C) K+ ions move out of the cell

D) K+ ions move into the cell

A

(C) Depolarization results from soidum ions moving into cells and repolarization from potassium moving out of the cell.

How well did you know this?
1
Not at all
2
3
4
5
Perfectly
29
Q

One of the reasons why the HIV virus selectively attacks CD4+ T lymphocytes as opposed to other cell types may be that:

A

The HIV virus displays the CCR5 receptor necessary for HIV to enter the cell.

How well did you know this?
1
Not at all
2
3
4
5
Perfectly
30
Q

Discuss the difference in mitochondria levels in a cancer cell and a normal cell.

A

A cancer cell has a higher metabolic rate than a normal cell, so it is expected to have a higher number of mitochondria to supply its energy. The cancer cell also shows a decreased time for its cell cycle as it reproduces faster.

How well did you know this?
1
Not at all
2
3
4
5
Perfectly
31
Q

During which phases of the cell cycle do mitochondria increase in number?

A

S, G1, and G2

How well did you know this?
1
Not at all
2
3
4
5
Perfectly
32
Q

Radioactive glucose will be absorbed the most by which of the following cell types?

A) Epithelial cell

B) Sarcomere

C) Pancreas cell

D) Schwann cell

A

(B) A sarcomere is a cell of the skeletal muscle system that needs large amounts of glucose for proper contraction.

How well did you know this?
1
Not at all
2
3
4
5
Perfectly
33
Q

Distinguish between the function of the smooth and rough endoplasmic reticulum.

A

Lipid production (smooth) and protein production (rough)

How well did you know this?
1
Not at all
2
3
4
5
Perfectly
34
Q

What is the site of the electron transport chain within the eukaryotic cell?

A

Inner mitochondrial membrane

How well did you know this?
1
Not at all
2
3
4
5
Perfectly
35
Q

Which of the following is correct about the human female reproductive cycle?

A) On day 14 of the menstrual cycle, an FSH surge occurs

B) The number of primary oocytes does not change after birth

C) Oogonia start dividing only after birth to create oocytes

D) Oocytes start dividing immeidately after birth to create oogonia

A

(B) Oocytogenesis is the process by which primordial follicles develop into oogonia which then divide between fertilization and birth to create primary oocytes. These begin mitosis but remain halted in prophase I. The process restarts at menarche, when a few of the oocytes continue development. On Day 14, a LH surge occurs, not FSH

How well did you know this?
1
Not at all
2
3
4
5
Perfectly
36
Q

Which of the following is an example of the skins role in the immune system?

A) Erector pili muscles controlling hair

B) Sensory nerve cells at various levels

C) Stored fat cells

D) Langerhans cells

A

(D) Langerhands cells are involved in processing microbial antigens

How well did you know this?
1
Not at all
2
3
4
5
Perfectly
37
Q

It turns out that alcoholics, who often have limited glycogen stores, can also suffer from ketoacidosis. Of note, the enzyme used to metabolize ethanol also reduces a molecule of NAD+ to NADH. Why might alcoholics be susceptible to ketoacidosis?

A

Ketoacidosis occurs as a result of increase ketone synthesis from Acetyl CoA. Increased ketone synthesis is stimulated by the absence of insulin in untreated Type I Diabetics. Impaired gluconeogenesis will result in low blood sugars and stimulate the hormone glucagon, which, in turn, will stimulate fatty acid oxidation that will result in accumulating Acetyl CoA and eventually ketone production.

How well did you know this?
1
Not at all
2
3
4
5
Perfectly
38
Q

Cyanide poisoning can be deadly. What is a plausible reason for why taking barbiturates is not toxic if it also inhibits part of the electron transport chain?

A

FADH2 can still donate electrons at Complex II with barbiturate poisoning

How well did you know this?
1
Not at all
2
3
4
5
Perfectly
39
Q

An inhibitor of Complex II would cause (three things):

A
  1. Impaired oxidative phosphorylation
  2. Potential damage to cells from reactive oxygen species
  3. A build-up of FADH2

Although Complex II doesn’t directly contribute to the proton gradient, it is decreasing electron flow to the downstream complexes (III and IV) that DO contribute to the protein gradient. Therefore, the proton gradient will decrease (it has to for ATP production to decrease as well). Note that a build-up in oxygen that normally is reduced by electrons in Complex IV will cause an increase in reactive oxygen species. Also note FADH2 can’t donate its electrons to Complex II anymore and therefore will not be re-oxidized to FAD.

How well did you know this?
1
Not at all
2
3
4
5
Perfectly
40
Q

A spirometer is used to measure a lung vital capacity at 4800 mL. Total lung capacity is estimated at 6000 mL. The difference between the two measurements reflects:

A) tidal volume

B) expiratory reserve volume

C) inspiratory reserve volume

D) residual volume

A

(D) Vital capacity is the maximum volume of air that can be moved into and out of the lungs. Residual volume refers to the amount of air left in the lungs after maximum exhalation. Total lung capacity is the vital capacity plus the residual volume.

How well did you know this?
1
Not at all
2
3
4
5
Perfectly
41
Q

A patient with osteoporosis would be expected to have high or low osteoclast and high or low osteoblast activity?

A

High osteoclast and low osteoblast activity because osteoblast cells deposit new bone whereas osteoclast cells reabsorb bone cells.

How well did you know this?
1
Not at all
2
3
4
5
Perfectly
42
Q

In a pancreatic abscess, all of the following would be expected to be found EXCEPT:

A) macrophages

B) viruses

C) bacteria

D) antigens on the surface of bacteria on walls

A

(B) An abscess results from tissue dying and becoming infected with bacteria. The macrophages of the immune system recognize the antigens on the bacteria cell wall and trigger endocytosis of the bacteria. Viruses need a living cell as a host and are not found in an abscess.

How well did you know this?
1
Not at all
2
3
4
5
Perfectly
43
Q

The result of oxygen and glucose deprivation to the brain is:

A) apoptotsis of presynaptic neurons due to high levels of Na+

B) apoptosis of postsynaptic neurons due to high levels of Ca2+

C) mitochondrial damage in postsynaptic neurons due to high intracellular glutamate

D) mitochondrial damage in presynpatic neurons due to high extracellular glutamate

A

(B) Oxygen deprivation results in disruption of the electrochemical gradient of the cell because of the failure of the sodium potassium pump. Glutamate transporters reverse direction, resulting in high levels of extracellular glutamate and triggering prolonged release of calcium ions into the post-synaptic neuron, leading to mitochondrial and apoptosis.

How well did you know this?
1
Not at all
2
3
4
5
Perfectly
44
Q

The electrochemical gradient of the cell is maintained at resting potential by:

A) simple diffusion of Na+ out of the cell

B) facilitated diffusion of Ca2+ into the cell

C) active transport of K+ out of the cell

D) active transport of Na+ out of the cell

A

(D) The sodium potassium pump requires ATP to pump Na+ out of the cell and K+ into the cell, thereby creating a net export of positive charge.

How well did you know this?
1
Not at all
2
3
4
5
Perfectly
45
Q

Some bacteria are small enough to live within a host cell, such as an epithelial cell in the small intestine. Suppose some bacteria were able to infect the cells in the ectoderm of an embryo. Which tissue or organ, in a newborn infant, would most likely be least damaged?

A) eyes

B) brain

C) small intestinal epithelial cells

D) hair follicles

A

(C). The ectoderm of an embryo cell will develop into the eyes, brain, and hair follicles. The endoderm develops into the digestive tract, including small intestinal epithelial cells.

How well did you know this?
1
Not at all
2
3
4
5
Perfectly
46
Q

In gastropod mollusks, a midgut gland situated on top of the stomach functions in secretion of digestive enzymes and in detoxification, absorption and excretion of the products of digestion and metabolism. These functions are analogous organs in mammals:

A) Spleen and kidneys

B) livers and pancreas

C) bile duct and sweat glands

D) stomach and small intestine

A

(B) The liver is responsible for detoxification of metabolic byproducts, glycogen storage, breakdown of red blood cells, and secretion of bile. The pancreas secretes digestive enzymes.

How well did you know this?
1
Not at all
2
3
4
5
Perfectly
47
Q

The population of bacteria in the small intestine may increase in the presence of an antibiotic if:

a) the bacteria are mutualistic
b) the pH of the small intestine decreases
c) the bacteria can undergo conjugation
d) the bacteria are not capable of sexual reproduction

A

(C) Conjugation is the transfer of genetical material from one bacteria to another and increases genetic variability.

How well did you know this?
1
Not at all
2
3
4
5
Perfectly
48
Q

Women infected with the HPV virus may develop cervical cancer. However, the probability of a woman developing colon cancer as a result of a bacterial infection in the colon is low because:

a) bacteria cannot undergo meiosis
b) bacteria can live inside a mammalian cell
c) bacteria do not contain oncogenes
d) bacteria do not directly insert or delete genes in the human genome.

A

Unlike viruses, bacteria do not insert genes directly into host cells.

How well did you know this?
1
Not at all
2
3
4
5
Perfectly
49
Q

A characteristic of fungi that is not shared with at least some animals is:

a) extracellular secretion of digestive enzymes
b) structural use of the polysaccharide chitin
c) haploid life cycle
d) asexual reproductoin

A

C. Animals have a haploid germ line, but it does not constitute a separate phase of the life cycle capable of independent reproduction.

How well did you know this?
1
Not at all
2
3
4
5
Perfectly
50
Q

Which of the following taxonomic categories would include taxa with the fewest nucleotide differences in rRNA sequence?

a) Domain
b) Phylum
c) Kingdom
d) Order

A

(D) The taxonomic categories are hierarchical, with more closely related organisms with the lowest sequence differences in the lower level taxa (order).

Kingdom | Phylum | Class | Order | Family | Genus | Species

How well did you know this?
1
Not at all
2
3
4
5
Perfectly
51
Q

Genetically modified λ bacteriophages are used as vectors to clone recombinant DNA by the process of:

a) transformation
b) transfection
c) transduction
d) conjugation

A

(C) Bacteriophages are viruses that infect bacteria so insertion of foreign DNA into a bacterium by a viral vector is called transduction.

How well did you know this?
1
Not at all
2
3
4
5
Perfectly
52
Q

GFR (glomerular filtration rate) measures the rate of filtrate passing from capillaries into the:

a) nephric duct
b) Bowman’s capsule
c) proximal convoluted tubules
d) ureter

A

(B) Water and solutes pass through the capillaries of the glomerulus into Bowman’s capsule

How well did you know this?
1
Not at all
2
3
4
5
Perfectly
53
Q

Which of the following would NOT be a consequence of low GFR?

a) electrolyte imbalance
b) changes in extracellular fluid volume
c) high levels of urea in urine
d) high blood creatinine

A

(C) Low filtration rate would result in lower levels of urea in the urine, not higher.

How well did you know this?
1
Not at all
2
3
4
5
Perfectly
54
Q

Malpighian tubules function as part of which organ system?

a) respiratory
b) circulatory
c) excretory
d) digestive

A

(C) These tubules remove nitrogenous waste from the body.

How well did you know this?
1
Not at all
2
3
4
5
Perfectly
55
Q

The fact that scar tissue on skin does not contain sweat glands can be explained how?

A

Differentiated cells have their fates determined and cannot be altered to generate another cell time. Sweat glands do result from differentiation of the ectoderm; but they cannot be formed from other differentiated cells.

How well did you know this?
1
Not at all
2
3
4
5
Perfectly
56
Q

Examinatiion of actively infested host cells with a light microscope would reveal:

a) viral DNA within the host cell nucleus
b) the remains of intracellular capsids
c) new virions budding from host cell membrane
d) no evident virions

A

D. Because of their small size, virions are not visible under a light microscope.

How well did you know this?
1
Not at all
2
3
4
5
Perfectly
57
Q

Viral mRNA is synthesized in the:

A

nucleus by RNA polymerase.

How well did you know this?
1
Not at all
2
3
4
5
Perfectly
58
Q

Which of the following would be involved in the cell-mediated immune response to combat cell-to-cell infection with a virus?

a) antibodies
b) immunoglobins
c) B cells
d) macrophages

A

(D). Macrophages and T-lymphocytes are components of the cell-mediated immune response.

How well did you know this?
1
Not at all
2
3
4
5
Perfectly
59
Q

The most likely target of host antibodies would be viral:

a) phospholipids
b) glycoproteins
c) DNA
d) capsid proteins

A

(B) Glycoproteins are virally-encoded proteins on the outer envelope of the virion which encloses the capsid. Phospholipids in the same envelope are derived from host nuclear membrane and would not elicit an immune response.

How well did you know this?
1
Not at all
2
3
4
5
Perfectly
60
Q

Synthesis of new viral glycoproteins would involve:

A

post-translational modifications in the lumen of the rough endoplasmic reticulum and Golgi body.

How well did you know this?
1
Not at all
2
3
4
5
Perfectly
61
Q

If the researcher is correct about increased albumin concentration in the interstitial fluid of the brain following focal ischemia (tissue dysfunction from reduced blood flow), which of the following interactions would most likely be affected?

A. Ependyma and cerebrospinal fluid

B. Microglia and lymphocytes

C. Schwann cells and axons

D. Astrocytes and blood vessels

A

(D) Schwann cells form the myelin sheath around axons of neurons in the peripheral nervous system. Microglia present antigens to lymphocytes. Astrocyte end feet surround blood vessels of the central nervous system to contribute to the blood brain barrier

How well did you know this?
1
Not at all
2
3
4
5
Perfectly
62
Q

Which cell type would most likely respond to brain tissue injury caused by focal ischemia by phagocytosis of debris from dead cells?

A. Ependymal cells
B. Microglia
C. Oligodendrocytes
D. Astrocytes

A

(B) The ependyma is a leaky barrier composed of ependymal cells between the cerebrospinal fluid and the interstitial fluid of the central nervous system. Oligodendrocytes form the myelin sheath around the axons of neurons in the central nervous system. Microglia secrete cytotoxic molecules, phagocytose cells and debris, and present antigens to immune cells.

How well did you know this?
1
Not at all
2
3
4
5
Perfectly
63
Q

Which cell type normally maintains homeostasis of the interstitial fluid in the brain, including the concentration of potassium ions?

A. Ependyma
B. Microglia
C. Neurons
D. Astrocytes

A

(D). The ependyma is a leaky barrier composed of ependymal cells between the cerebrospinal fluid and the interstitial fluid of the central nervous system. Neurons process and transmit information in the nervous system. Astrocytes maintain homeostasis of the interstitial fluid of the central nervous system.

How well did you know this?
1
Not at all
2
3
4
5
Perfectly
64
Q

Which cell type most likely removes damaged myelin in multiple sclerosis?

A) Ependyma

B) Microglia

C) Oligodendrocytes

D) Astrocytes

A

(B) Microglia phagocytose debris in the central nervous system, therefore they most likely remove damaged myelin in multiple sclerosis.

How well did you know this?
1
Not at all
2
3
4
5
Perfectly
65
Q

If the researcher is correct about a virus infecting cells of the central nervous system leading to loss of myelin by immune attack on these cells in multiple sclerosis, which cell type would most likely be infected?

A. Microglia
B. Neurons
C. Oligodendrocytes
D. Astrocytes

A

(C) Oligodendrocytes form the myelin sheath around axons of neurons in the central nervous system; therefore they would be the cells most likely to be infected if the hypothesis is correct.

How well did you know this?
1
Not at all
2
3
4
5
Perfectly
66
Q

The scientist transiently opens sodium channels on a dendrite of a cultured neuron. Which membrane potential term best describes what most likely happens to the membrane around the open channel during this event?

A

Since the membrane potential went from -60 mV to -50 mV, this indicates that depolarization is occurring.

How well did you know this?
1
Not at all
2
3
4
5
Perfectly
67
Q

Gigantism, a disease of accelerated growth, can be caused by primary tumors of the anterior pituitary gland. In patients with tumor-induced gigantism, levels of Growth Hormone Releasing Hormone would expected to be:

A

The hypothalamus would respond to increased levels of GH by reducing production of GHRH.

How well did you know this?
1
Not at all
2
3
4
5
Perfectly
68
Q

Hormones that stimulate their target tissue via the intermediate endocrine glands are categorized as:

A

Tropic hormones. The tropic hormones are those with other endocrine glands as their target. They are mostly produced and secreted by the anterior pituitary gland and include the hormones FSH, LH, ACTH, and TSH.

How well did you know this?
1
Not at all
2
3
4
5
Perfectly
69
Q

Describe how a graph would look like if it were to measure hormone concentration using a radioimmunoassay to measure hormones.

A

Initially, hormone concentration is low and radioactivity is high, but radioactivity decreases as hormone concentration is elevated.

How well did you know this?
1
Not at all
2
3
4
5
Perfectly
70
Q

Low serum levels of Iodine, a necessary component to the production of thyroid hormones would lead to which of the following:

I. goiter

II. hypothyroidism

III. elevated serum TSH

A

All three of them. The question stem states that Iodine is necessary in the production of thyroid hormones. Insufficient levels of iodine would result in decreased thyroid hormone production. When thyroid hormone levels are low, the thyroid gland is stimulated to increase production by the anterior pituitary hormone TSH (Thyroid Stimulating Hormone). Hypothyroidism is a medical condition characterized by low circulating levels of the thyroid hormones T3 (triiodothyronine) and T4 (thyroxine). When thyroid hormone levels are low, the anterior pituitary gland releases the hormone TSH in order to stimulate the thyroid gland to increase production of its hormones. This can result in thyroid tissue growth (goiter) to compensate for decreased efficacy.

How well did you know this?
1
Not at all
2
3
4
5
Perfectly
71
Q

Increased production of cortisol in response to stress would most likely accompany increased:

A. bone formation

B. constriction of pupils

C. blood pressure
D. secretion of digestive enzymes

A

C. Cortisol is produced in the cortex (outer layer) of the adrenal gland which is situated atop the kidneys. During times of stress, the corticotropin-releasing hormone (CRH) from the hypothalamus triggers adrenocorticotropic hormone (ACTH) from the anterior pituitary to be produced, and this in turn stimulates cortisol production in the adrenal glands. Cortisol has many effects on the body. One important effect is that it increases blood pressure by increasing the sensitivity of the vasculature to epinephrine and norepinephrine

How well did you know this?
1
Not at all
2
3
4
5
Perfectly
72
Q

Discuss the pathway of blood from being deoxygenated to becoming oxygenated

A

DEOXYGENATED

capillaries –> venules -> veins -> superior/inferior vena cava -> right atrium -> right venticle -> pulmonary artery -> lungs

OXYGENATED

Lungs –> pulmonary vein -> left atrium -> left ventricle -> aorta -> arteries -> arterioles -> capillaries

How well did you know this?
1
Not at all
2
3
4
5
Perfectly
73
Q

Where does blood entering the right atrium come from? A. Left atrium
B. Right ventricle
C. Pulmonary circulation
D. Systemic circulation

A

(D) Systemic circulation captures the blood flow that occurs through the arteries, capillaries, and veins that are between the left ventricle and the right atrium. Therefore blood entering the right atrium comes from the systemic circulation.

How well did you know this?
1
Not at all
2
3
4
5
Perfectly
74
Q

Why would it be unlikely for a clot formed in the venous system to cause a heart attack or stroke?

A. Venous blood is less likely to clot due to its lower oxygen concentration.

B. A clot formed in the venous system would get trapped in the lungs before getting into systemic circulation.

C. Venous clots are rare because pressure in the venous system is so low.

D. Clots fall apart as they travel through the blood.

A

(B) The venous system (systemic circulation) is a low-pressure system that carries blood with low oxygen concentration. Relative to the arterial system, clot formation in the venous system is not a particularly “rare” event. Blood from the venous system (systemic circulation) moves towards the right side of the heart. If a blood clot formed in the venous system it would move through veins into the superior or inferior vena cava and then into the right atrium, right ventricle, and the lungs. A clot needs to be in the arterial side of the systemic circulation to cause a heart attack or stroke. When a blood clot goes into the pulmonary circulation, it will often get stuck in pulmonary arteries, or arterioles, or capillaries, and hence never reach the arterial side of the systemic circulation.

How well did you know this?
1
Not at all
2
3
4
5
Perfectly
75
Q

Which of the following would you NOT expect to increase systolic blood pressure?

A. Taking a medication that Increases fluid retention in the kidneys
B. Experiencing a nightmare
C. Taking a medication that causes vasoconstriction
D. Changing from lying down to standing up

A

(D). Systolic blood pressure will increase with increased blood flow or resistance. Vasoconstriction will cause increased resistance. Fluid retention will cause increased blood flow (volume/time) due to increased total blood volume. Experiencing a nightmare will raise your heart rate. Heart rate multiplied by stroke volume equals blood flow. Therefore a nightmare will increase blood flow. Changing from lying to standing up will decrease blood pressure because gravity would cause blood to “pool” in lower extremity venous system. This reduces the blood flow because the “pool” of blood in the legs is not effectively circulating.

How well did you know this?
1
Not at all
2
3
4
5
Perfectly
76
Q

Which of the following is true regarding the coronary arteries?

A. The coronary arteries carry blood to the heart from the lungs.
B. The coronary arteries carry blood to the lungs from the heart.
C. The coronary arteries are part of systemic circulation.
D. All blood from systemic circulation passes through the coronary arteries before going back to systemic circulation.

A

(C). The lungs are connected to the heart by the pulmonary arteries and veins (pulmonary circulation), as well as bronchial arteries and veins (systemic circulation). The coronary arteries come off the base of the aorta, and are the first branches off of the aorta. Just like other tissues in the body, the heart needs to be perfused by blood to get oxygen and clear wastes. The blood in the atria and ventricles do not fulfill this purpose because they do not divide into capillary beds.

How well did you know this?
1
Not at all
2
3
4
5
Perfectly
77
Q

Which of the following are most important in explaining how the body changes the resistance of arterioles to blood flow?

A. Endothelial cells
B. Smooth muscle cells
C. Skeletal muscle fibers
D. Arterial valves

A

(B) Arterial walls can be opened (vasodilation) or closed (vasoconstriction) to change the internal diameter of the vessel. Valves do not play an important role in changing the size of the blood vessel. Contraction of smooth muscle cells will cause arteriole vasoconstriction and will increase resistance to blood flow.

How well did you know this?
1
Not at all
2
3
4
5
Perfectly
78
Q

How might blood flow to the kidneys during excerise decrease?

A

During exercise blood needs to get redistributed to the areas doing work (skeletal muscles). Vessels that are wide open (vasodilated) offer less resistance to blood flow, then vessels that are narrow (vasoconstricted). During exercise, blood vessels in skeletal muscle dilate to offer more blood to those areas. In addition, blood vessels going to other organs (e.g. kidney) constrict.

How well did you know this?
1
Not at all
2
3
4
5
Perfectly
79
Q

Over which of the following blood vessel types is the drop in blood pressure typically greatest?

A. Arterioles

B. Arteries

C. Capillaries

D. Venules

A

(A). If you take someone’s blood pressure, typically you measure the pressure in their brachial artery. It does not significantly matter how far along the brachial artery you measure. Pressures in the venous system are generally low and do not drop significantly. One arteriole divides into many capillaries, and hence much more blood needs to force through an arteriole than a capillary. More blood pressure is needed to force blood through arterioles than capillaries, even though one arteriole would have lower resistance than one capillary. As a result, the drop in blood pressure is greatest along the arterioles.

How well did you know this?
1
Not at all
2
3
4
5
Perfectly
80
Q

Partial pressure of gas X in inspired air: 18 mmHg

Inspirational volume: 700 mL

Partial pressure of gas X in expired air: 6 mmHg

What is the residual volume in the lungs?

A

The total volume in the lungs after inspiration is x + 700 mL, if x is the residual volume. Therefore the partial pressure in the expired gas will be diluted by a factor (x + 700 mL) / 700 mL.

The residual volume will be 1400.

How well did you know this?
1
Not at all
2
3
4
5
Perfectly
81
Q

What is a result of the fact that we have residual volume in our lungs?

A

Air we breathe in has oxygen, while residual volume will mostly have given its oxygen to the blood..

Oxygenated air we breathe in is diluted by de-oxygenated air already in the lungs

How well did you know this?
1
Not at all
2
3
4
5
Perfectly
82
Q

On inspiration, will where gas X go after the trachea?

A

The order is mouth → trachea → bronchi → bronchioles → alveoli

How well did you know this?
1
Not at all
2
3
4
5
Perfectly
83
Q

Vasoconstriction of which of the following vessels will most effectively reduce fat absorption from the small intestine into the bloodstream?

A) Lacteals inside intestinal villi of the small intestine

B) Capillaries in the smooth muscle of the small intestine

C) Lacteals in the peritoneum around the small intestine

D) Capillaries in the peritoneum around the small intestine

A

(A). If the lacteals of the intestinal villi were to contract, it would reduce fat absorption because ingested facts collect in the lacteals for transport into the venous circulation.

How well did you know this?
1
Not at all
2
3
4
5
Perfectly
84
Q

An intravenous infusion causes a sharp rise in the serum level of albumin which will most likely cause an:

a) increase in the immune response
b) increase in tissue albumin levels
c) outflow of blood fluid to the tissues
d) influx of tissue fluid to the bloodstream

A

(D) The plasma proteins can not cross the walls of the blood vessels, but water molecules can. The walls of the artery acts as a semipermeable membrane setting up the conditions needed for osmosis to occur. An increase in plasma albumin will upset the osmotic balance because blood will become hypertonic (with respect to the tissue). So water will have to flow into the bloodstream to reestablish equilibrium. One of the causes of edema, increased fluid in body tissues, is a decrease in the plasma protein level.

How well did you know this?
1
Not at all
2
3
4
5
Perfectly
85
Q

A lower than normal blood pressure will cause which of the following effects on the rate of plasma clearance?

A) an increase because the concentration of a substance in the urine wil increase

B) an increase because ADH levels will be very low

C) a decrease because the decreased rate of urine output will allow more reabsorption by the kidney

D) a decrease because ADH levels will be very high

A

(C) Low blood pressure could have an effect on ADH levels, but there’s not enough information to decide. The best answer is C. Low blood pressure DECREASES the glomerular filtration rate, allowing more time for reabsorption and decreasing the amount of a certain substance in the urine.

How well did you know this?
1
Not at all
2
3
4
5
Perfectly
86
Q

What would happen to the blood in the glomerular capillaries if the heart stopped?

A

Fluid in the space around the glomerulus would flow back into the capillary bloodstream because the protein-rich blood would be hypertonic with respect to the protein-poor fluid in the capsular space so that the fluid would flow down the osmotic gradient into the blood.

How well did you know this?
1
Not at all
2
3
4
5
Perfectly
87
Q

The increased blood pressure resulting from higher than normal concentration of ADH most likely affected the urinary output of a substance by increasing the:

A

Glomerular filtration rate. Increasing blood pressure should increase the flow of fluid through the kidney system and decrease, rather than increase, water reabsorption.

How well did you know this?
1
Not at all
2
3
4
5
Perfectly
88
Q

Postmenopausal women receiving estrogen and progesterone therapy will most likely experience which of the following side effects?

A) breast tissue will atrophy

B) vaginal tissue will dry out

C) periodic menstruation will resume

D) lactation will be induced

A

(C) Estrogen and progesterone are actively secreted by the ovaries of premenopausal women and act to maintain the uterine cycle. With advancing age, the ovary becomes less responsive to pituitary gonadotropins and cyclical changes in the endometrium of the uterus disappear. The menstrual cycle can be restablished by administration of estrogen and progesterone in a regiment that approximates the rise and fall of hormone levels in pre-menopausal women.

How well did you know this?
1
Not at all
2
3
4
5
Perfectly
89
Q

Production of which of the following hormones will be inhibited by the administration of dietary calcium to prevent osteoporosis?

A) growth hormone

B) calcitonin

C) thyroid hormone

D) parathyroid hormone

A

(D) Calcium levels in the blood need to be kept constant. Parathyroid hormone and calcitonin regulate blood levels of calcium. Calcitonin puts calcium in the bones. Parathyro_id_ hormone gets rid of calcium from the bones.

How well did you know this?
1
Not at all
2
3
4
5
Perfectly
90
Q

Which of the following describes the solubility of methionine?

A) High in both water and lipids.

B) High in water and low in lipids.

C) Low in water and high in lipids.

D) Low in both water and lipids.

A

(C) Methionine is a non-polar, hydrophobic amino acid. Water is a polar solvent, and lipids are nonpolar. Thus, methionine will have low solubility in water and high solubility in lipids.

How well did you know this?
1
Not at all
2
3
4
5
Perfectly
91
Q

Most serum homocysteine exists in a form bound to other thiol amino acids and proteins in the form of disulfides (e.g. cystine-homocystine and homocystine-homocystine). Disulfide bonds play a role in which levels of protein structure?

A

Tertiary & Quarternary structure. Primary structure is determined by covalent peptide bonds. Secondary structure is determined by hydrogen bonding between distance side chains of amino acids in the same polypeptide. Disulfide bonds play a role in determining tertiary and quaternary structure.

How well did you know this?
1
Not at all
2
3
4
5
Perfectly
92
Q

RNA interference (RNAi) is a mechanism by which short RNA sequences bind onto specific targets of RNA molecules. One of its consequences is the targeting of RNA molecules for intracellular destruction. This protects cells against viral and other infectious RNA molecules. In this capacity, RNAi’s function is analogous to which of the following?

A macrophages

B cytotoxic T cells

C antibodies

D apoptosis

A

(C) Antibodies are proteins which bind to antigens, or structures the immune system has targeted. The antibody-antigen relationship is highly specific, with antibodies generally targeting a specific site on the antigen. Antigens that have been bound by antibodies are easier to capture and destroy by the immune system. This is analogous to the function of RNAi wherein RNA sequences bind onto specific target RNA sequences for destruction like infections molecules.

How well did you know this?
1
Not at all
2
3
4
5
Perfectly
93
Q

Which of the following is always true concerning the base composition of DNA?

A) in each single strand, the number of adenine residues equals the number of thymine residues

B) in each single strand, the number of adenine residues equals the number of guanine residues

C) in a molecule of double stranded DNA, the ratio of adenine residues to thymine residues equals the ratio of cytosine residues to guanine residues

D) in a molecule of double stranded DNA, the number of adenine residues plus thymine residues equals the number of cytosine residues plus guanine residues

A

(C) Since A always binds with T and C always binds with G, the A/T ratio and the ratio of G/C is equal to one.

How well did you know this?
1
Not at all
2
3
4
5
Perfectly
94
Q

A molecule of DNA contains all of the following EXCEPT:

a) deoxyribose sugars
b) polypeptide bonds
c) phosphodiester bonds
d) nitrogenous bases

A

(B) DNA is a nucleotide polymer. A nucleotide is a ribose sugar, phosphate group, and a nitrogenous base. The nucleotides in DNA are held together by phospodiester bonds.

How well did you know this?
1
Not at all
2
3
4
5
Perfectly
95
Q

Which of the following is a carbohydrate polymer that is stored in plants and digestible by animals?

a) starch
b) glycogen
c) cellulose
d) glucose

A

(A) Plants store carbohydrates as starch. Animals store carbohydrates as glycogen. Glucose is not a polymer. Cellulose is found in plant cell walls and is not digestible by humans.

How well did you know this?
1
Not at all
2
3
4
5
Perfectly
96
Q

Proline is not technically an alpha amino acid. Due to the ring structure of proline, it cannot conform to the geometry of the alpha helix, and creates a bend in the polypeptide chain. This phenomenon asists in the creation of what level of protein structure?

A) primary

B) secondary

C) tertiary

D) quaternary

A

(C) The bending of the polypeptide chain is the tertiary structure of a protein.

How well did you know this?
1
Not at all
2
3
4
5
Perfectly
97
Q
A
How well did you know this?
1
Not at all
2
3
4
5
Perfectly
98
Q
A
How well did you know this?
1
Not at all
2
3
4
5
Perfectly
99
Q

Translation in eukaryotic cells is associated with each of the following organelles EXCEPT:

a) mitochondrial matrix
b) cytosol
c) nucleus
d) rough endoplasmic reticulum

A

(C) Translation does not occur in the nucleus.

How well did you know this?
1
Not at all
2
3
4
5
Perfectly
100
Q

How many chromosomes does a human primary spermatocyte contain?

A) 23

B) 46

C) 92

D) 184

A

(B) A primary spermatocyte has finished the S stage of interphase but not the first meiotic division.
Thus, it has 46 chromosomes

How well did you know this?
1
Not at all
2
3
4
5
Perfectly
101
Q

A scientist monitors the nucleotide sequence ofthe third
chromosome as a cell undergoes normal meiosis. What is
the earliest point in meiosis at which the scientist can
deduce with certainty the nucleotide sequence of the third
chromosome of each gamete?
A. prophase I
B. metaphase I
C. prophase II
D. telophase II

A

(A). In normal meiosis, the only change in the nucleotide sequence of the third chromosome will occur
during crossing over. Crossing over occurs in prophase 1.

102
Q
A

(D) In metaphase I we see tetrads.

103
Q

Which of the following characterizes mitotic prophase?
A. chromosomal alignment along the equator of the
cell
B. separation of sister chromatids
C. centriole migration to the cell poles
D. cytokinesis

A

(C) Centrioles migrate in prophase of mitosis. Chromosomes align in metaphase; centromeres split in
anaphase; cytokinesis usually occurs during telophase.

104
Q

All of the following might describe events occurring in
prophase I of meiosis EXCEPT:
A. tetrad formation
B. spindle apparatus formation
C. chromosomal migration
D. genetic recombination

A

(C) In prophase I, a tetrad will form.and genetic recombination will occur; spindle apparatus will always form; BUT chromosomal migration describes anaphase

105
Q

When a human female is born, the development of her
oocytes is arrested in:
A. prophase of mitosis
B. prophase I of meiosis
C. prophase II of meiosis
D. interphase

A

(B) The life cycle of all oocytes is arrested at the primary oocyte stage until puberty

106
Q

Which of the following events does NOT playa role in
the life cycle of a typical retrovirus?
A. Viral DNA is injected into the host cell.
B. Viral DNA is integrated into the host genome.
C. The gene for reverse transcriptase is transcribed and
the mRNA is translated inside the host cell.
D. Viral DNA incorporated into the host genome may
be replicated along with the host DNA.

A

(A) A retrovirus contains RNA which is reverse transcribed to DNA and then incorporated into the
host cell genome.

107
Q

A mature virus outside the host cell is called a virion. A
virion may contain all ofthe following EXCEPT:
A. a capsid.
B. an envelope made from a phospholipid bilayer.
C. core proteins.
D. both RNA and DNA

A

(D) A virus cannot contain both DNA and RNA. Many viruses contain proteins.

108
Q
A

(A) In the lysogenic cycle of viral infection is a cell that harbors inactive viral DNA in its genome.

109
Q

Which of the following structures are found in
prokaryotes?
I. A cell wall containing peptidoglycan.
II. A plasma membrane lacking cholesterol.
III. Ribosomes
A. I only
B. II only
C. I and II only
D. I, II, and 1lI

A

(D) Prokaryotes have a cell wall that contains peptidoglycan, ribosomes, and a plasma membrane
without cholesterol.

110
Q

DNA from phage resistant bacteria is extracted and
placed on agar with phage-sensitive E. coli. After
incubation it is determined that these E. coli are now also
resistant to phage attack. The most likely mechanism for
their acquisition of resistance is:
A. transduction.
B. sexual reproduction.
C. transformation.
D. conjugation.

A

(C)

  • Because DNA is acquired directly from the medium, this is transformation
  • There is no sexual reproduction in bacteria
  • Transduction is the transfer of DNA via a virus
  • Conjugation occurs between two viruses
111
Q

Bacteriophages are parasites that infect bacterial cells in
order to carry on their life function. When a phage
transfers bacterial DNA from one host to another this
process is called:
A. transformation.
B. transduction.
C. conjugation.
D. transmission.

A

(B) Transduction is the transfer of DNA via a virus.

112
Q

A staphylococcus infection is most likely caused by an
organism that is:
A. rod-shaped
B. spherical
C. a rigid helix
D. a non-rigid helix

A

(B)

  • Bacilli are rod-shaped;
  • Spirilli are rigid helixes;
  • Spirochetes are not rigid;
  • Cocci are round.
113
Q
A

(B) . Fungi are heterotrophs, not autotrophs,

114
Q

The Kingdom of Fungi is divided into:
A. phyla
B. divisions
C. orders
D. species

A

(B) Similar to the plant kingdom the fungi kingdom is divided into divisions.

115
Q

All of the following are composed of microtubules
EXCEPT:
A. the tail of a sperm cell
B. the spindle apparatus
C. the cilia of the fallopian tubes
D. the flagella of bacteria

A

(D) The flagella of bacteria are made from the protein flagellin.

116
Q

Which ofthe following is true concerning the nucleolus?
A. It is bound by a phospholipid membrane.
B. It disappears during prophase.
C. It is the site oftranslation of ribosomal RNA.
D. It is found in most bacteria.

A

(B) The nucleolus is the site of rRNA transcription not translation. It is not membrane bound and
should not be confused with the nucleoid of prokaryotes.

117
Q
A
118
Q

Which of the following is not a membrane bound
organelle?
A. The golgi body
B. The nucleus
C. The smooth endoplasmic reticulum
D. The ribosome

A

(D) Ribosomes are made of RNA and protein. They do not have a phospholipid bilayer

119
Q

Which of the following is found in vertebrates but NOT
in invertebrates?

A) a dorsal, bollow nerve chord

B) mylenation to increase the speed of nervous impulse
transmission along the axon

C) axons through which the nerve impulse is conducted

D) Na/K pump

A

(B) Invertebrates do not have myelinated axons to accelerate nervous impulse transmission. Instead,
they rely upon increased size. Invertebrate is a subphylum of Chordate which is characterized by a dorsal nerve
chocd at some point in their development.

120
Q

Which of the following activities is controlled by the cerebellum?

A) involuntary breathing movements

B) fine muscular movements during a dance routine

C) contraction of the thigh muscles during the knee jerk reflex

D) absorption of nutrients across the microvilli of the small intestine

A

(B) The cerebellum controls finely coordinated muscular movements. Involuntary breathing movements are controlled by the medulla oblongata and the knee-jerk reflex is governed by the spinal cord

121
Q

If an acetylcholine antagonist were administered generally into a person, all of the following would be affected EXCEPT:

a) the neuroeffector synapse in the sympathetic nervous system
b) the neuroeffector synapse in the parasympathetic nervous system
c) the neuromuscular junction of the somatic nervous system
d) the ganglionic synapse in the sympathetic nervous system

A

(A) Every type of synapse in the peripheral nervous system uses acetylcholine as its neurotransmitter
except the second (the neuroeffector) synapse in the sympathetic nervous system. You may not have known
what a neuroeffector synapse was, but you should have been able to reason that it is an end-organ synapse. An
effector is an organ or a muscle, something that responds to neural innervation by making something happen
in the body.

122
Q

Pressure waves in the air are converted to neural signals
at the:
A. retina
B. tympanic membrane
C. cochlea
D. semicircular canals

A

(C) . Pressure waves, or sound, are converted to neural signals by hair cells in the organ of
Corti in the cochlea.

123
Q

Reflex arcs:

A. involve motor neurons exciting the spinal cord dorsally

B. require fine control by the cerebral cortex.

C. always occur independently of the central nervous
system.
D. often involve inhibition as well as excitation of
muscle groups.

A

(D) In order to prevent conflicting contractions by antagonistic muscle groups, reflexes will often cause one muscle group to contract while it sends an inhibitory signal to the antagonistic muscle group. Motor neurons exit ventrally from the spinal cord. Reflex arcs (at least somatic ones) are usually confined to the spinal cord; they do not require fine control by the cerebral cortex. This eliminates B. Reflex arcs may be integrated by an interneuron in the spinal cord. C is out as well.

124
Q

Which part of the brain controls higher-level thought
processes?
A. the thalamus
B. the cerebellum
C. the cerebrum
D. the medulla

A

(C) This is a knowledge based question. The cerebrum is also called the cerebral cortex.

125
Q

A cook touches a hot stove and involuntarily withdraws
his hand before he feels pain. Which of the following
would not be involved in the stimulus-response pathway
described?
A. a neuron in the cerebellum
B. a neuron in the spinal cord
C. a motor neuron
D. a sensory neuron

A

(A) The question describes a simple reflex arc which does not involve neurons in the brain.

126
Q
A
127
Q

Which of the following is true for all endocrine
hormones?
A. They act through a second messenger system.
B. They bind to a protein receptor.
C. They dissolve in the blood.
D. They are derived from a protein precursor.

A

(B) All hormones bind to a protein receptor, whether at the cell membrane, in the cytoplasm, or in the
nucleus of the cell. Steroids and thyroxine require a transport protein to dissolve in the aqueous solution of the
blood. Steroids are derived from cholesterol, not protein precursors.

128
Q

All of the following act as second messengers for
hormones EXCEPT:
A. cyclic AMP
B. calmodulin
C. acetylcholine
D. cyclic GMP

A

(C) Acetylcholine acts through a second messenger system, and is not a second messenger itself.

129
Q

Vasopressin, a hormone involved in water balance, is
produced in the:
A. hypothalamus.
B. posterior pituitary.
C. anterior pituitary.
D. kidney

A

(A) The hormones of the posterior pituitary are synthesized in the bodies of neurons in the hypothalamus, and transported down the axons of these nerves to the
posterior pituitary

130
Q

A drug that causes increased secretion of testosterone from the interstitial cells of a physically mature male would most likely:
A. cause the testes to descend prematurely.
B. delay the onset of puberty.
C. cause enhanced secondary sex characteristics.
D. decrease core body temperature.

A

(C)

131
Q

The heart, bone and skelelal muscle most likely arise
from which ofthe following primary germ layers?
A. The ectoderm
B. The endoderm
C. The gastrula
D. The mesoderm

A

(D) Generally, the inner lining ofthe respiratory and digestive tracts, and associated organs, come from
the endodenn. The skin, hair, nails, eyes and central nervous system come from ectoderm. Everything else
comesfrom the mesodenn. The gastrula is not a germ layer.

132
Q

Which of the following does NOT describe cleavage in
human embryos?
A. The solid ball of cells produced during cleavage is
called a morula.
B. The size o fthe embryo remains constant throughout
the cell divisions of cleavage.
C. Cell division occurs in one portion of the egg in
meroblastic cleavage.
D. Daughter cells are genetically identical to parent
cells

A

(C) Mammalian eggs undergo holoblastic cleavage where division occurs throughout the whole egg.
At first glance, this question appears to ask for somewhat obscure knowledge about meroblastic cleavage.
However, you should be able to eliminate A, B, and D quite easily as being part of human embryonic cleavage,
so it is unnecessary to know meroblastic or holoblastic cleavage

133
Q

Which of the following is responsible for the production of tesosterone

A) anterior pituitary

B) pancreas

C) adrenal cortex

D) adrenal medulla

A

(C) The adrenal cortex makes many other steroid based hormones, as well as testosterone.

134
Q
A
135
Q
A
136
Q
A
137
Q
A

(C) Pepsin, whose optimum pH is around 2.0, denatures in the environment of the small intestine,
whose pH is between 6 and 7. A is wrong because pepsin isn’t working at all in the small intestine; it won’t be
working synergistically with trypsin. B is wrong because pepsinogen is activated in the stomach by low pH. D
is wrong because pepsin is a catalyst, which makes it a protein; amylase digests starch.

138
Q

Which of the following is the best explanation for why pancreatic enzymes are secreted in zymogen form?

A) delay in digestion is required in order for bile to increase the surface area chyme

B) enzymes are most active in zymogen form

C) zymogens will not digest bile in the pancreatic duct

D) pancreatic cells are not as easily replaced as intestinal epithelium

A

(D) A only applies to lipases, B is false, and C is true but not an adequate explanation.

139
Q

One function of the large intestine is:
A. to absorb water
B. to secrete excess water
C. to digest fat
D. to secrete urea

A

(A) The large intestine absorbs water. Fat is digested in the small intestine. Urea is secreted in the kidney.

140
Q

All of the following enzymes are part of pancreatic
exocrine function EXCEPT:
A. bile
B. chymotrypsin
C. pancreatic amylase
D. lipase

A

(A) Pancreatic exocrine function includes enzymes made in the pancreas and secreted through a duct.
Bile is not an enzyme. It is made in the liver and stored in the gallbladder.

141
Q

In humans, most chemical digestion offood occurs in the:
A. mouth
B. stomach
C. duodenum
D. ilenm

A

(C) . Most chemical digestion occurs in the first part of the small intestine, the duodenum.

142
Q

A stomach ulcer may increase the acidity ofthe stomach.
The stomach cells most affected by a stomach ulcer are:
A. goblet cells.
B. parietal cells.
C. chief cells.
D. G cells

A

(B) Parietal cells secrete HCl. Goblet cells secrete mucus. Chief cells secrete pepsin. G cells secrete gastrin into the cell.

143
Q
A

(D) Most fat digestates enter the lymph as chylomicrons via lacteals. Smooth endoplasmic reticulum
synthesizes triglycerides.

144
Q
A

(A) Essential’ means that the body cannotsynthesize them. Nonessential amino acids are synthesized
by the liver. Amino acids are absorbed by facilitated and active transport. Urea is the end product of amino acid
deamination in the liver.

145
Q

Most of the glycogen in the human body is stored in the
liver and the skeletal muscles. Which of the following
hormones inhibits glycogenolysis?
A. Cortisol
B. Insulin
C. Glucagon
D. Aldosterone

A

(B) Insulin lowers blood sugar. One way is through inhibiting glycogenolysis

146
Q

Essential amino acids must be ingested because they
cannot be synthesized by the body. In what form are these
amino acids likely to enter the blood stream?
A. single amino acids
B. dipeptides
C. polypeptides
D. proteins

A

(A) Macromolecules are broken down to their most basic units.

147
Q
A

(C) The only process available for the removal of wastes by the Bowman’s capsule is diffusion, aided
by the hydrostatic pressure of the blood.

148
Q
A
149
Q
A

(B) High blood pressure would result in more fluid being forced into Bowman’s capsule

150
Q
A

(A) The loop of Henle concentrates the medulla via a net loss of solute to the medulla. This process is
critical to the function of other parts of the nephron; a medulla with a high concentration of solute allows for
the passive absorption of water from the filtrate in other areas of the nephron.

151
Q

Which of the following is responsible for the spread of
the cardiac action potential from one cardiac muscle cell
to the next?
A. gap junctions
B. desmosomes
C. tigbt junctions
D. acetylcholine

A

(A) The cardiac action potential is spread from one cardiac muscle cell to the next via ion movement
through gap junctions.

152
Q

In the congenital heart defect known as patent ductus
arteriosus, the ductus arteriosus, which connects the aorta and the pulmonary arteries during fetal development, fails to close at birth. This will likely lead to all of the following EXCEPT:
A. equal, or increased, oxygen concentration in the
blood that reaches the systemic tissues.
B. increased oxygen concentration in the blood that
reaches the lungs.
C. increased work load imposed on the left ventricle.
D. increased work load imposed on the right ventricle.

A

(A)

153
Q
A
154
Q
A
155
Q
A
156
Q

Sustained heavy exercise results in all of the following
changes to blood chemistry except:
A. lowered pH
B. raised CO2 tension
C. increased temperature
D. decreased carboxyhemoglobin

A

(D) Heavy exercise manifests increased carbon dioxide production that leads to increased carboxyhemoglobin

157
Q

Humoral immunity involves the action of:
A. cytotoxic T lymphocytes.
B. stomach acid.
C. pancreatic enzymes.
D. immunoglobulins.

A

(D) Cytotoxic T cells work in cell mediated immunity. Stomach acid plays a
role in the nonspecific innate immunity; humoral immunity is specific and acquired. Immunoglobulins, or antibodies, are involved in the humoral immunity system.

158
Q

Which of the following would you not expect to find in a
lymph node?
A. B lymphocytes
B. proteins discarded by tissue cells
C. invading bacteria
D. old erythrocytes

A

(D) Old erythrocytes are destroyed in the spleen and the liver.

159
Q

An individual exposed yo a pathogen for Ihe first time will
exhibit an innate immune response involving:
A. B lymphocytes
B. T lymphocytes
C. granulocytes
D. An individual exposed to a pathogen for the first
time must acquire immunity before it can respond.

A

(C) The innate immune response does not involve humoral immunity (B-cell) or cell mediated immunity (T-cell). The innate immune system responds to any and every foreign invader with the white blood cells
called granulocytes as well as with inflammation and other actions.

160
Q

Which of the following is true concerning B negative type blood?

A) Type B negative blood will make antibodies that attack type A antigens but not type B antigens

B) Type B negative blood will make antibodies that attack type B antigens but not type A antigens

C) Type B negative blood will make antibodies that attack only type O antigens

D) Type B negative blood will make antibodies that attack both type A antigens and type O antigens

A

(A) Type B negative blood carries B antigens and not the Rh factor. It does not carry A antigens. There are no O antigens. Thus, B negative blood makes antibodies that will only attach to A antigens and Rh antigens.

161
Q

During a muscular contraction:
A. both the thin and thick lilaments contract.
B. the thin filament contracts. but the thick filament
does not.
C. the thick filament contracts, but the thin filament
does not.
D. neither the thin nor the thick filament contract.

A

(D) Neither actin (the thin filament) nor myosin (the thick filament) changes its length during muscular contraction; instead, the proportion of actin and myosin overlap increases

162
Q

When undergoing physical exercise, healthy adult
skeletal muscle is likely to respond with an increase in all
ofthe following except:
A. glycolysis
B. lhe Citric Acid Cycle
C. mitosis
D. protein production

A

(C) All muscles will need more energy and protein if they are being used rigorously, but in humans,
mature skeletal muscle cells do not divide.

163
Q

The biceps muscle is connected to the radius bone by:
A. biceps tendon
B. annular ligament ofthe radius
C. articular canilage
D. the triceps muscle

A

(A) MCAT doesn’t test anatomy. This question is asking if you know that tendons connect muscle to
bone. Ligaments connect bone-to-bone. Tendons are not cartilage.

164
Q

Skeletal muscle contraction may assist in all of the
following EXCEPT:
A. movement of fluid through the body
B. body temperature regulation
C. posture
D. peristalsis

A

(D) D is correct. Peristalsis is a function ofsmooth muscle only. Shivering is an example of temperature regulation
by skeletal muscle. Skeletal muscle may assist in venous blood movement and lymph fluid movement.

165
Q

Which of the following muscular actions is controlled by
the autonomic nervous system?
A. the knee-jerk reflex
B. conduction of cardiac muscle action potential from
cell to cell
C. peristalsis ofthe gastrointestinal tract
D. contraction ofthe diaphragm

A

(C) Peristalsis is a smooth muscle activity and smooth muscle is innervated by the autonomic nervous system. The skeletal muscles are innervated by the somatic nervous system, so interruption of the autonomic nervous system would not affect the knee-jerk reflex or the diaphragm. An action potential in cardiac muscle is conducted from cell to cell by gap junctions.

166
Q
A
167
Q
A
168
Q
A
169
Q
A
170
Q
A
171
Q

Surgical cutting of which of rbe following tissues would
result in the LEAST amount of pain’?
A. muscle
R. bone
C. cartilage
D. skin

A

(C) Cartilage does not have nerves

172
Q

In a synovial joint, the connective tissue holding the
bones together are called:
A. ligaments
B. tendons
C. muscles
D. osseous tissue

A

(A) Ligaments attach bone to bone. Tendons attach bone to muscle.

173
Q

All ofthe following are functions of bone EXCEPT:
A. mineral storage
B. structural support .
C. blood temperature regulation
D. fat storage

A

(C) Bone does not regulate the body or blood temperature. It does store calcium and phosphate, support and protect the body, produce blood cells, and store fat

174
Q

All of the following are found in compact bone EXCEPT:
A. yellow marrow
B. haversian canals
C. canaliculi
D. Volkmann’s canals

A

(A) Yellow bone marrow is usually found in the medullary cavity of long bones.

175
Q

Hydroxyapatite is the mineral portion of bone.
Hydroxyapatite contains all of the following elements
except:
A. calcium
B. sulfur
C. phosphate
D. hydrogen

A

(B) Hydroxyapatite is made up of calcium and phosphate in a compound that includes hydroxyl groups as well. You should know that bone acts as a storage place for phosphate and calcium

176
Q

The spongy bone of the hips is most important in:
A. red blood storage
B. red blood cell synthesis
C. fat storage
D. lymph fluid production

A

(B) Spongy bone contains the blood stem cells important for blood cell synthesis. Fat storage is in long bones. Red blood storage is responsible in the liver and spleen.

177
Q

All of the following factors would most likely favor an
r-selection reproductive strategy over a K-selection
strategy EXCEPT:
A. intense seasonal droughts
B. a short growing season
C. limited space
D. large scale commercial predation by humans

A

(C) r-strategy is more efficient when the prevailing conditions are governed by density independent
factors such as harsh environment, short seasons, etc. Conunercial predation methods have a similar effect. r-strategists strategists are able to better withstand massive predation from man or others, because they produce so many more offspring than they need to continue the species. K-strategists fare better when the prevailing conditions
are governed by density dependent factors such as limited resources.

178
Q

Which of the following statements is FALSE?

A) Both eukaryotes and prokaryotes contain the electron transport chain

B) The ribosome of a prokaryote is 70S and the ribosome of a eukaryote is 80S

C) Prokaryotes only have one RNA polymerase and eukaryotes have three RNA polymerases

D) Both eukaryotes and prokaryotes process mRNA between transcription and translation

A

(D) Only eukaryotes process their mRNA between transcription and translation—this is why they cannot be in the same location.

179
Q

A researcher has an agar plate covered with a lawn of E. Coli. She adds a drop of a substance, and the next day, there is a clear spot on the plate where the substance was added. The substance could be:

I. A virus undergoing the lytic cycle

II. A virus undergoing the productive cycle

III. A chemical that is toxic to prokaryotes

A

(I and III) An addition of a lytic virus or a toxin will kill E. coli. Animal viruses only go through the productive cycle because viruses cannot bud out of a cell with a cell wall, such as bacteria.

180
Q

Which of the following statements concerning viruses is true?

A) The productive cycle is the most efficient ineffective cycle for phages

B) Viruses that infect human cells must have an envelope

C) Genetic information can be transferred between hosts via transfection

D) A virus with an RNA genome must code for an RNA dependent RNA polymerase

A

(D) In order to replicate its genome, an RNA virus must code for an RNA-dependent RNA polymerase; this enzyme will create a new strand of RNA by reading a template strand of RNA. Viral host cells will not express these enzymes naturally; they have no need to make RNA by reading RNA. Host cells normally producing RNA using DNA as a template.

181
Q

Discuss the direction of COP-I proteins and COP-II proteins.

A

COP-I proteins: cis Golgi to rough endoplasmic reticulum

COP-II proteins: rough endoplasmic reticulum to cis Golgi, medial Golgi, and then trans Golgi and then to the cell membrane.

182
Q

What is the correct order for the signals leading to the formation of cyclic AMP given epinephrine stimulation?

A

Epinephrine -> G-Protein Coupled Receptor -> G-proteins -> adenylyl cyclase -> cAMP

183
Q

What is pleiotropism?

A

Alteration of a gene affects many different aspects of a phenotype. Example: mutation of a gene affects heart, bone, and inner ears

184
Q

What is polygenism?

A

Complex traits that are influenced by many different genes. Example: height is influenced by genes for growth factors, hormones, receptors, bone deposition, etc.

185
Q

What is epistasis?

A

Expression of alleles for one gene is dependent on a different gene. Example: A gene for curly hair cannot be expressed if a different gene causes baldness.

186
Q

What is the maximal frequency of recombination?

A

When there is no linkage and the genes are assorted independently. There would be 50% maximal frequency of recombination.

187
Q

How is recombination frequency calculated?

A

Number of recombinant phenotypes / total number of progeny

188
Q

Does recombination occur between genes more frequently if they are near each other or far apart?

A

The farther the two genes are away from each other, the greater the odds that recombination will occur between them

189
Q

What is the correct order of events during synaptic transmission?

A
  1. Action potential reaches the end of an axon at the synaptic knob
  2. Depolarization of presynaptic membrane
  3. Voltage-gated calcium channels open
  4. Neurotransmitter is released from presynaptic cell
  5. Neurotransmitter crosses the synaptic cleft
  6. Neurotransmitter binds to ligand-gated ion channels on the post synaptic membrane
  7. Membrane depolarization of postsynaptic cell
  8. Voltage-gated sodium channels open
  9. An action potential is initiated
  10. Neurotransmitter in the synaptic cleft is degraded and/or removed
190
Q

High blood pressure could be rectified through:

I. an increase in ADH

II. a decrease in aldosterone

III. ACE inhibitors

A

II and III. An increase in ADH leads to increased retention of water, increased blood volume, and increased blood pressure. Aldosterone causes increased sodium absorption at the distal tubule to reabsorb water which triggers the release of ADH. ACE (angiotensin converting enzyme) converts angiotensin I to angiotensin II, a very powerful vasoconstrictor and stimulator of aldosterone release

191
Q

The cardiac cycle is divided into systole and diastole during which the ventricles contract and relax, respectively. During systole, blood is ejected from the ventricles and is pumped into either the aorta or the pulmonary artery. The pressure of the ventricles rapidly increases, however the volume remains unchanged. This is referred to as isovolumetric contraction. What is the most likely reason for isovolumetric contraction?

A

The pressure in the aorta is initially greater than that of the left ventricle. At the start of systole, the semilunar valves are closed and although the ventricles are contracting, no blood is ejected. The semilunar valves remain closed until the pressure within the ventricles exceeds that of the aorta or pulmonary artery at which the semilunar valves open and blood is ejected from the ventricles.

192
Q

The presence of the ductus arteriosus as a result of failure to close after birth results in what:

A

Increased vascular pressure in the pulmonary circulation because the mixing of deoxygenated and oxygenated blood would result in an increase in blood volume and a subsequent increase in pulmonary vascular pressure.

193
Q

T-cell antigen receptors are different from antibodies because:

A) T-cell receptors must interact with antigen uniquely presented by other but not with free antigen

B) T-cell receptors bind various cytokines

C) T-cell receptors bind complement proteins to lyse cells

D) T-cell receptors are mediators of allergic reactions

A

(A) T-cell receptors must bind antigen that is presented by MHC proteins on the surface of other cells. They are unable to bind free antigens. Cytokines have their own family of receptors and T-cell receptors do not bind complement proteins.

194
Q

Which of the following statements about antibodies is true?

I. Antibodies are produced by B cells

II. Antibodies only recognize peptide antigen

III. Antibodies can mark an antigen for destruction by killer T cells.

A

Only I. Antibodies are produced by the B cells of the immune system. Antibodies can recognize a great diversity of antigens. Killer T cells do not directly destroy antigens, rather, they kill cells that are producing antigens.

195
Q

The loop of Henle uses a countercurrent multiplier system to create an area of high ion concentration in the renal medulla. Which limb is permeable to ions and which limb is permeable to water?

A

The descending limb of the loop of Henle is permeable to water, but not to ions. The ascending limb of the loop is permeable to ions, but not water.

196
Q

All of the following would increase glomerular filtration rate EXCEPT:

A) constriction of efferent arterioles in the kidney

B) increased circulatory volume

C) dilation of afferent arterioles in the kidney

D) increased filtrate osmolarity

A

(D) Constricting efferent arterioles would restrict flow out of the glomerulus and dilating afferent arterioles increase flow into the glomerulus; in either case, pressure at the glomerulus would rise, and filtration rate would increase. An increase in volume will increase in pressure. If the filtrate osmolarity is high, the macula densa will not send signals to stimulate the juxtaglomerular cells and the GFR will not be increased.

197
Q

Is angiotensin II a vasoconstrictor or vasodilator?

A

Vasoconstrictor. It increases blood pressure

198
Q

Is angiotensin II a substrate for ACE?

A

No. Angiotensin I is, and it is converted by ACE to Angiotensin II.

199
Q

The hepatic portal vein carries deoxygenated blood from the alimentary canal into the liver, where it is filtered before continuing to the heart via the hepatic vein and inferior vena cava. Blood from which of the following organs does NOT drain into the hepatic portal vein?

A) stomach

B) small intestine

C) colon

D) kidneys

A

The stomach, small intestine, and colon (large intestine) are part of the alimentary canal and their blood supplies drain into the hepatic portal vein to be filtered by the liver. Blood that passes through the kidneys drains directly into the inferior vena cava via the renal veins.

200
Q

Which of the following is NOT true regarding smooth muscle cells?

A) They are uninucleate and relatively small in size compared to skeletal muscle

B) Gap junctions may exist between adjacent smooth muscle cells

C) Ca2+ influx and binding to troponin results in actin and myosin binding and contraction

D) They lack striations due to nonlinear alignment of the sarcomeres

A

(C) Smooth muscle is activated very differently from cardiac or skeletal muscle. Smooth muscle lacks troponin and tropomyosin and utilizes myosin light-chain kinase. All the other statements are true.

201
Q

Rigor mortis, the stiffening of body limbs after death, is due to:

A) the inability of the myosin head to detach from actin due to loss of ATP

B) the inability of the myosin head to attach to actin due to the loss of ATP

C) the inability of the myosin head to detach from actin due to the loss of ADP

D) the inability of the myosin head to attach to actin due to the loss of ADP

A

(A) During muscle contraction, the binding of ATP to the myosin head is required to detach myosin from the actin filament.

202
Q

Which of the following is a possible way to distinguish between muscle types?

A) Skeletal muscle can contract via peristalsis, while smooth muscle cannot

B) Skeletal muscle contraction depends on motor neuron stimulation, while cardiac muscle does not

C) Cardiac muscle cells are syncytial, while smooth muscle cells are not

D) Cardiac muscle contractions are regulated by influxes of calcium, while smooth muscle is not

A

(B) It is true that skeletal muscle contraction depends on motor neuron stimulation. Cardiac muscle is not voluntary; contraction is controlled by the sinoatrial node. Smooth muscle contracts via peristalsis. Cardiac muscle cells form a function syncytium, the cells are connected by gap junctions and function as unit; smooth muscle also works like this. All muscle contractions are regulated by calcium

203
Q

A patient presents to the emergency room in diabetic ketoacidosis, a life-threatening complication of diabetes, which results in metabolic acidosis, as well as very high serum glucose levels. Which of the following vital signs would be expected in this patient, as compared to baseline?

A) Increased blood pressure

B) Increased respiratory rate

C) Decreased heart rate

D) Decreased respiratory rate

A

(B) The metabolic acidosis (decreased blood pH) would lead to hyperventilation in an attempt to compensate for the low pH by reducing the amount of CO2. DKA leads to high levels of serum glucose to overwhelm the glucose reabsorption mechanisms of the kidney, leading to glucose in the urine. This glucose leads to potentially severe dehydration to lower blood pressure

204
Q

What is the most likely explanation for why patients with asthma have difficulty breathing?

A) Constriction of bronchial smooth muscle leads to diminished air flow

B) Dilation of bronchial smooth muscle leads to diminished air flow

C) Constriction of smooth muscle surrounding blood vessels leads to decreased blood flow to the lungs

D) Interference with skeletal muscle neurons leads to decreased force of contraction of the diaphragm

A

(A) In asthma, constriction of bronchial smooth muscles decreases the diameter of the bronchi, leading to diminished air flow. Sympathomimetics stimulate the adrenergic receptors on smooth muscle, causing the muscle to dilate and improve air flow through the bronchi.

205
Q

A middle-aged man is in a coma after a motor vehicle accident. His respiratory rate has significantly decreased due to trauma to the medulla. All of the following are true EXCEPT:

A) there will be a decrease in the [H+] of his blood

B) normal regulation of ventilation rate has been disrupted

C) there will be an increase in PCO2 of his blood

D) there will be a decrease in PO2 of his blood

A

(A) The reducing rate of respiration will lead to accumulation of CO2 and a drop of O2 in the blood. The excess CO2 will shift the equilibrium to increase [H+] and drop in pH. The normal feedback mechanisms (such as pH and pressure) will not be effective in restoring ventilatory patterns.

206
Q

Which of the following structures undergoes mitosis?

A) Spermatid

B) Spermatogonium

C) Primary spermatocyte

D) Secondary spermatocyte

A

(B) The spermatogonia in the testes periodically undergo mitosis to produce both more spermatogonia and primary spermatocytes. Primary spermatocytes, which are diploid, undergo meiosis I to become haploid secondary spermatocytes, which then undergo meiosis II to become haploid spermatids

207
Q

Which of the following statements regarding childbirth is true:

A) Release of oxytocin from the anterior pituitary, combined with increased mechanical pressure of the fetal head on the cervix, creates a positive feedback loop that increases uterine contraction

B) Release of progesterone from the placenta, combined with increased mechanical pressure of the fetal head on the cervix, creates a positive feedback loop that increases uterine contractions

C) Release of progesterone from the posterior pituitary combined with increased mechanical pressure of the fetal head on the cervix, creates a negative feedback loop that increases uterine contractions

D) Release of oxytocin from the posterior pituitary, combined with mechanical pressure of the fetal head on the cervix, creates a positive feedback loop that increases uterine contractions

A

(D) Labor and delivery is one example of a positive feedback loop where the release of oxytocin from the posterior pituitary combined with mechanical pressure of the fetal head work together to increase uterine contractility in an effort to expel the baby from the uterus. Progesterone, released early in pregnancy from the corpus luteum and later in pregnancy from the placenta itself, decreases uterine contractility. Levels of progesterone are high early in pregnancy in order to keep the developing fetus inside the uterus, but levels diminish later in pregnancy in anticipation of the upcoming delivery of the baby.

208
Q

Which of the following is NOT a difference between spermatogenesis and oogenesis?

A) Spermatogenesis in a male begins at puberty whereas oogenesis in a female begins when the female is an embryo

B) Spermatogenesis produces four sperm whereas oogenesis produces one ovum

C) Spermatogenesis produces primary spermatocytes for a male’s entire life, whereas oogenesis ceases to produce primary oocytes when a female reaches menopause

D) Spermatogenesis occurs in the testes whereas oogenesis occurs in the ovaries

A

(C) Spermatogenesis only begins once a male reaches puberty, and then continues for the rest of his life. Oogenesis begins when a female is an embryo and the cells are arrested at the primary oocyte phase when she is a fetus. Spermatogenesis produces four sperm from each spermatogonia whereas oogenesis produces one ovum and two polar bodies per oogonia. The testes are the male gonads and the site of spermatogenesis; the ovaries are the female gonads and the site of oogenesis.

209
Q

Which of the following hormones is NOT elevated during the first trimester of pregnancy?

A) Estrogen

B) Progesterone

C) GnRH

D) hCG

A

(C) The corpus luteum secretes estrogen and progesterone, which helps maintain pregnancy (these are elevated for the first trimester). Estrogen and progesterone feedback inhibit the secretion of GnRH from the hypothalamus. hCG is a hormone secreted by the embryo that helps to maintain the corpus luteum during the first trimester until the placenta is formed.

210
Q

Ovulation usually occurs on the 14th day of the ovarian cycle. All of the following occurs during ovulation and the days immediately following ovulation EXCEPT:

A) the ovary releases a secondary oocyte

B) a surge of FSH from the anterior pituitary causes the follicle to become the corpus luteum

C) the follicle secretes progesterone once it becomes the corpus luteum

D) a surge of LH can be detected

A

(B) During ovulation, the ovary releases a secondary oocyte. A surge of LH, not FSH from the anterior pituitary, causes the follicle to become the corpus luteum. Once the follicle becomes the corpus luteum, it produces and secretes progesterone that stabilizes and enhances the endometrium.

211
Q

Postpartum women often experience mild to moderate uterine contractions when nursing. These contraction are triggered by the release of:

A) estrogen

B) oxytocin

C) progesterone

D) prolactin

A

(B) When suckling occurs, oxytocin is released from the posterior pituitary to trigger milk ejection from the glands towards the nipple. As this hormone is also responsible for stimulating uterine contractions during delivery, it can have a similar but less intense effect when nursing. Prolactin is involved in nursing, but is responsible for stimulating the production of milk and does not have an effect on the uterus.

212
Q

The greater vestibular glands in the female (Bartholin’s glands) have a similar function as which of the following male reproductive glands

A) Bulbourethral glands (Cowper’s glands)

B) Seminal vesicles

C) Prostate

D) Testes

A

(A) The greater vestibular glands are located at the posterior of the vaginal opening, are stimulated on arousal and secrete an alkaline. This helps neutralize the acidity of the vagina to make it a more hospitable environment for sperm, which can be damaged by acids. The bulbourethral glands in the male are stimulated on arousal and secrete an alkaline mucus into the urethra. This helps neutralize any traces of acid that might remain from earlier passage of urine through that duct, and makes the urethra a more hospitable environment for sperm. The seminal vesicles and the prostate produce semen and are stimulated at orgasm and the testes are the male primary sex organs; they produce sperm and testosterone.

213
Q

Presumably, hyperglycemia promotes cellular dehydration because:

A) glucose, as an energy source, accelerates the osmotic work performed by plasma membranes

B) glucose, as an energy source, accelerates plasma membrane ion exchange pumps

C) glucose molecules raise the osmotic pressure of the extracellular space

D) glucose molecules are exchanged for water molecules across the plasma membrane

A

(C) Glucose release from glycogen stores raises the osmotic pressure of body fluids and dehydrates cells by protecting them from freezing. The glucose acts as an antifreeze, lowering the freezing point of the frog’s body fluids. Freezing is gradual in that water moves osmotically out of cells into the concentrated unfrozen extracellular fluid. Ice formation can thereby be restricted to extracellular water.

214
Q

A bacterium containing genes for sex pilus construction gave rise to a daughter cell lacking these genes. This most probably occurred because:

A) the bacterial chromosome was not completely replicated prior to cell division

B) the cell membrane failed to move the replicated chromosomes apart

C) copies of the plasmid containing the genes were not equally distributed to the new daughter cells.

D) one copy of the plasmid containing the genes was digested by a bacterial lysosome

A

(C) Plasmids are not attached to the cell membrane and may not be equally distributed among daughter cells in asexual reproduction. Building of a cytoplasmic bridge is directed by plasmid genes rather than by chromosomal genes. Therefore, neither chromosomal replication nor chromosomal segregation is directly pertinent to sex-pilus construction. Bacterially generally lack membrane enclosed organelles such as lysosomes.

215
Q

What occurs in the gastrointestinal tract when a patient has a ruptured appendix?

A

The appendix is ruptured, and since it is continuous with the colon, the bacteria in the colon can move between the abdominal cavity from the appendix.

216
Q

The two primary factors that normally determine the level of blood pressure are:

A) the blood concentration of L-NMMA and norepinephrine

B) the cardiac output and the resistance to blood flow

C) the blood volume and the amount of L-arginine in the diet

D) the heart rate (heartbeats/min) and the cardiac stroke volume

A

(B) Cardiac output (stroke volume x heart rate) determines the amount of blood pumped into the system by the heart per unit time. The resistance to blood flow is determined by the caliber of the small arteries, arterioles, and precapillary sphincters. The blood pressure equals total peripheral resistance times cardiac output, a relationship analogous to Ohm’s Law for electrical circuits.

217
Q

Most fungal spores are:

A) metabolically active and diploid

B) metabolically inactive and haploid

C) relatively sensitive to environmental changes

D) encased in a porous nuclear membrane

A

(B) Although diploid spores do exist, most fungal cells are haploid. Spore cells are not metabolically active, and are not encased in a porous nuclear membrane.

218
Q

Septic shock can be caused by the release of cytokines. To reduce the mortality rate of septic shock, a treatment plan was proposed: administration of an antibody that blocks the attachment site for the endotoxin-binding protein complex on macrophages

For this treatment to be effective, the antibody must:

A) stimulate macrophage activation

B) stimulate T-cell production

C) inhibit T-cell production

D) inhibit macrophage activation

A

(D) By blocking the proliferation of macrophages, their ability to release cytokines is blocked. Binding of the antibody to the macrophage does not stimulate its development and cytokine release.

219
Q

In the macronucleus, the genes for rRNA are located extrachromosomally. This suggests that the rRNA genes in this bacteria are:

a) nonlinear
b) nonfunctional
c) self-replicating
d) rearranged

A

(C) Genes are strands of DNA so this must mean that they are strands of DNA that are not part of the cell’s chromosomes. These strands still must self-replicated just as chromosomes do if they are passed from one generation to the next.

220
Q

The macronuclei of the asexual progeny in Tetrahymena and the cytoplasm of the ova-producing cells of female vertebrates share a common feature in that both:

A) undergo uneven division

B) contain uneven amounts of nuclear material

C) regulate their contents by adding or skipping an S phase

d) are apportioned at mitosis

A

(A) When a vertebrate oocyte divides in meiosis, the cytoplasm is distributed to two daughter cells. Most of it goes to the daughter cell destined to be the ovum. The other daughter cells, the polar bodies, are cast off with little cytoplasm. The amount of nuclear material in each, however, is the same.

221
Q

Contraction of the diaphragm results in a:

A) more negative interpleural pressure and inspiration

B) more negative interpleural pressure and expiration

C) more positive interpleural pressure and inspiration

D) more positive interpleural pressure and expiration

A

(A) The diaphragm is a muscular partition between the abdominal and thoracic cavities. It is dome-shaped at rest, curving up toward the lungs and heart. It flattens when it contracts during inspiration. Because it is anchored around its edges to the ribs and spines, when the diaphragm contracts, the volume of the thoracic cavity increases which decreases the interpleural pressure in the pleural cavity between the thoracic wall and the lung. Fresh air flows in to equalize the pressure inflating the lung.

222
Q

If an artery that supplies blood to a lung lobe was blocked, but ventilation to the lobe was unaffected, how would alveolar gas partial pressure change?

A) Both PO2 and PCO2 would increase

B) Both PO2 and PCO2 would decrease

C) PO2 would increase and PCO2 would decrease

D) PO2 would decrease and PCO2 would increase

A

(C) If the blood flow to an alveolus were blocked, there would be no flow of hemoglobin rich red blood cells to take away O2 and no influx of CO2 from the blood. As a result, the air would be more like the atmosphere: it would have a higher PO2 and a lower PCO2

223
Q

One would expect that the development of striated muscle in arthropods was paramount to this group’s success because such tissue would allow for:

A) increased activity

B) a muscular digestive system

C) a pumping circulatory system

D) the development of a coelom

A

(A) Striated muscle is very efficient at turning potential energy in the chemical bonds of glucose into kinetic energy of muscle contraction. It is striated because the contractile proteins are packed into dense arrays to maximize power and minimize bulk.

224
Q

After observing the well-defined nervous system of several insect species, a student concludes that all arthropods must have a true coelom. Is such a conclusion warranted?

A) No, insects are not representative of all arthropods

B) No, coelom development and nerve tissue development are independent processes

C) Yes, only a true coelom gives rise to nerve tissue

D) Yes, both the coelom and nerve tissue are derivatives of mesoderm

A

(B) The embryonic process by which a coelom or body cavity forms is distinct from the process by which the nervous system forms. In both arthropods and vertebrates the coelom forms as pouches in the mesoderm. The vertebrate central nervous system arises from the ectoderm.

225
Q

If a person’s gallbladder is removed, the person should restrict the consumption of:

A) proteins

B) polysaccharides

C) triglycerides

D) lactose

A

(C) The gall bladder is an organ that stores bile produced by the liver. The major dissolved components of bile are breakdown products of hemoglobin such as bilirubin and bile salts. The bile salts are amphipathic and are soluble in fats and oils and in water. Bile salts allow for dietary fats and oils to form an emulsion of tiny droplets dispersed in the digestive juices. Triglycerides are hydrophobic fats, so bile would aid in their digestion.

226
Q

Which of the following statements explains most plausibly why host antibodies are ineffective against H. pylori?

A) antibody proteins may be denatured in the harsh environment of the stomach

B) antibodies are not generally effective against bacteria

C) H. pylori infection may suppress the activity of the immune system

D) Antibodies are not secreted from host tissues into extracellular spaces

A

[A] stomach ulcers and some forms of gastric cancer may be linked to H. pylori infections in the stomach. Without treatment by antibiotics, such infections can be persistent. Antibody proteins may be denatured in the harsh environment of the stomach.

227
Q

Which of the following compounds could be described by this graph?

A

(B). It has 9 hydrogens and 1 unique hydrogen.

228
Q

Normally the immune system avoids attacking the tissues of its own body because:

a) a special intracellular process recognizes only foreign antigens
b) the body does not make any antigens that the immune system could recognize
c) it changes its antibodies to be specific only to foreign antigens
d) it suppresses cells specific to the body’s own antigens

A

[D] the immune system is designed to attack foreign material in the body. It avoids attacking tissues of its own body because it suppresses cells that are specific to its own body’s antigens (surface molecules that would otherwise initiate an immune response)

229
Q

Embryonic mouse cells divide every 10 hours at 37ºC. How many cells would be produced from an egg after three days?

A) Fewer than 50

B) Between 50 and 500

C) Between 500 and 5000

D) More than 5000

A

[B] x cells would be present after 3 days (72 hours). The number can be calculated by tracking the doubling time: 1 to 2 to 4 to 8 to 16 to 32 to 64 to 128 to 256. At the end of 70 hours (approximately 3 days), there would be 128 cells, assuming that each cell underwent division 10 hours after its previous division.

230
Q

The concentration of the protein cyclin rises and falls during the cell cycle as shown in Figure 1:

What mechanism could account for this oscillation of cyclic protein concentration?

A) replication of the cyclin gene during S phase of interphase

B) segregation of chromosomes carrying the cyclin genes during mitosis

C) translation of cyclin mRNA in interphase and proteolysis on cyclin protein in mitosis

D) translation of cyclin mRNA in mitosis and proteolysis of cyclic protein in interphase

A

[C] The graph shows that the concentration of cyclin rises and falls in a regular manner throughout the cell cycle, reaching a peak just at the beginning of mitosis, gradually declining during mitosis, reaching a minimum at the end of the mitosis, and gradually increasing during interphase. The mechanism that can best account for this oscillation in the concentration of cyclin is translation of cyclin mRNA (creating the ptoerin from mRNA template) followed by proteolysis of cyclic protein during mitosis.

231
Q

In mammals, which of the following events occurs during mitosis but does NOT occur during meiosis I?

A) synapsis

B) splitting of centromeres

C) pairing of homologous chromosomes

D) breaking down of nuclear membrane

A

[B] Their anaphases are different. In mitosis, sister chromatids are pulled apart at the centromeres, each becoming an independent chromosome in two diploid cells. During anaphase I, homologous pairs are separated into two daughter cells but each chromosome still has two sister chromatids joined to each other at the centromere.

232
Q

Dewlaps that reflect UV light would evolve by natural selection only if:

A) individuals with UV-reflective dewlaps produced more offspring than did individuals without them

B) individuals with UV-reflective dewlaps were better able to communicate than individuals without them

C) individuals with UV-reflective dewlaps were less subject to predation than individuals without them

D) individuals with UV-reflective dewlaps mated more frequency than did individuals without them

A

[A] The organism needs to produce offspring, passing on the genes that cause the advantageous phenotype.

233
Q

Uric acid is formed in the breakdown of purines to xanthine, a uric acid precursor. What nitrogenous base would promote the formation of uric acid crystals in gout?

A) Cytosine

B) Uracil

C) Guanine

D) Thymine

A

[C] The passage states that uric acid is formed by the breakdown of purines. Guanine is one type of purines found in cells. So is adenine.

234
Q

Why do calcium supplements often include Vitamin D?

A) Vitamin D is needed to prevent rickets

B) The activated form of vitamin D stimulates the absorption of calcium into the blood

C) The activated form of vitamin D enhances the action of calcitonin

D) The activated form of vitamin D enhances the uptake of calcium by bone tissue

A

[B] Activated vitamin D acts on the small intestine to stimulate the absorption of calcium into the bloodstream. The inclusion of vitamin D in calcium supplements would ensure that vitamin D is present in the body to help promote this absorption.

235
Q

All of the following occur during normal inspiration of air in mammals except:

A) elevation of the rib cage

B) relaxation of the diaphragm

C) reduction of pressure in the pleural cavity

D) contraction of the external intercostal rib muscles

A

[B] The diaphragm contracts and pulls downward, causing air to enter the lungs. Elevation of the rib cage increases the volume inside the chest cavity, reduction of pressure inside the pleural cavity causes air to move into the lungs, and contraction of the external intercostal rib muscles helps the chest expand.

236
Q

If oligonucleotides such as mRNA were not degraded rapidly by intracellular agents, which of the following processes would be most affected?

A) the production of tRNA in the nucleus

B) the coordination of cell differentiation during development

C) the diffusion of respiratory gases across the cell membrane

D) the replication of DNA in the nucleus

A

[B] The destruction of mRNA prevents continuous protein production, allowing cell to change its cell differentiation during development.

237
Q

An effective and efficient method for the delivery of an antisense gene could be:

A) orally as an emulsified product

B) microinjection into individual body cells

C) intravenously as a nonantigenic, blood-stable product

D) infection of an embryo by a virus modified to carry the gene

A

[D] By infecting an embryo with a virus, the virus could be incorporated into the genome of the embryonic cells thus causing all cells derived from these embryonic cells to contain the antisense gene.

238
Q

Which of the following nucleotide sequences describes an antisense molecule that can hybridize with the mRNA sequence 5’-CGAUAC-3’?

A) 5’-GCTATG-3’

B) 5’-GCUAUG-3’

C) 3’-GCUAUG-5’

D) 3’-GCAUAG-5’

A

[C] The 3’ end lines up with the 5’ end of the complement.

239
Q

To be an effective therapy, an antisense gene that is incorporated into a genome that contains the target gene must be:

A) on the same chromosome as the target gene but not necessarily be physically adjacent

B) on the same chromosome as the target gene and must be physically adjacent

C) regulated in a similar manner as the target gene

D) coded on the same stand of DNA as the target gene

A

[C] The antisense gene needs to be regulated in a manner similar to the target gene so it would be produced at the same time that the sense mRNA is produced to prevent translation.

240
Q

Capillaries in the kidney and elsewhere in the body maintain fluid homeostasis by balancing hydrostatic and osmotic pressures. Which of the following is the initial effect of a blood clot forming on the venous side of a capillary bed?

A) net fluid flow in the direction of interstitial spaces will increase

B) net fluid flow in the direction of interstitial spaces will decrease

C) capillary osmotic pressure will increase

D) capillary osmotic pressure will decrease

A

[A] After introducing the idea of fluid homeostasis which is by balancing hydrostatic and osmotic pressures, we need to know that blood flows from arteries to capillaries and to veins. So if flow is blocked on the venous side, blood would accumulate in the capillaries which would make hydrostatic pressure building up in the capillaries, causing a net increase in fluid flow in interstitial spaces

241
Q

The normal path of sperm movement from the male testis to the point of fertilization in the female is:

A) epididymis, vas deferens, urethra, vagina, cervix, uterus, fallopian tube

B) epididymis, vas deferens, uréter, cérvix, uterus, fallopian tuve

C) epididymis, vas deferens, urethra, vagina, uterus, ovary

D) interstitial cells, epididymis, vas deferens, vagina, uterus, ovary

A

[A] As sperm cells leave the testis, they travel through the epididymis to the vas deferens and into the urethra. The sperm then enter the female’s vagina, travel through the cervix and uterus, and enter the fallopian tube where fertilization most commonly takes place.

242
Q

Which of the following processes is LEAST directly influenced by adrenergic drugs?

A) Peristalsis

B) Secretion of digestive enzymes

C) Enzymatic breakdown of food molecules

D) Nutrient delivery to muscles and organs

A

[C] Adrenergic drugs mimic activation of the sympathetic nervous system; the best answer is the process LEAST directly controlled by the sympathetic nervous system. It inhibits peristalsis and secretion of digestive enzymes, and causes dilation of blood vessels. The sympathetic nervous system does not affect activity of digestive enzymes after they have been secreted.

243
Q

Which participant in the electron chain transport chain has the greatest attraction for electrons?

A) FAD

B) NAD+

C) Oxygen

D) Cytochrome C

A

[C] The final electron acceptor of this chain, oxygen, has the greatest attraction for electrons. FADH2 and NADH have more of an affinity for electrons than their counterparts. Cytochrome c is slightly more electronegative, but oxygen is more.

244
Q

Muscles with striated fibers are the primary muscle type in:

A) the heart

B) the uterus

C) arteries and veins

D) the small intestine

A

[A] See attachment.

245
Q

Accumulation of DDT in the testes may cause reduced fertility in males because the uncoupling of oxidative metabolism from ATP production may reduce:

A) glucose concentration of semen

B) testosterone concentration of semen

C) blood circulation in the testes

D) sperm motility

A

[D] Sperm motility requires large amounts of ATP as evidenced by the high concentration of mitochondria in the sperm midpiece.

246
Q

Where in the human male reproductive system do the gametes become motile and capable of fertilization?

A) Testis

B) Urethra

C) Epididymis

D) Prostate Gland

A

[C] Upon entering the epididymis, spermatozoa lack motility and the ability to fertilize ova. They acquire these abilities during their transit in the epididymis.

247
Q

Which of the following best describes the bond that would form between the following two nucleotides if they were located adjacent to each other as shown in a single strand of DNA?

A) A bond between the phosphate of the thymine and the phosphate of the adenine

B) A bond between an oxygen in the thymine base and a nitrogen in the adenine base

C) A bond between the phosphate of the thymine and the sugar of the adenine

D) A bond between the phosphate of the adenine and the sugar of the thymine

A

[D] The most likely bond would be between the phosphate of the bottom nucleotide (adenine) and the sugar of the top nucleotide (thymine). This is the familiar bonding pattern that makes up the backbone of DNA: alternating sugar and phosphate groups joined by ester bonds.

248
Q

Assume that a certain species with sex chromosomes R and S exists such that RR individuals develop as males and RS individuals develop as females. Which of the following mechanisms would most likely compensate for the potential imbalance of sex-chromosome gene products between males and females of this species?

A) Inactivation of one R chromosome in males

B) Doubling transcription from the S chromosome in females

C) Inactivation of the R chromosome in females

D) Doubling transcription from the R chromosome in males

A

[A] From the two X chromosomes in the female is reduced by randomly inactivating one of the X chromosomes that if the male is homozygous, inactivation of one R chromosome in males would achieve the appropriate reduction in output of genetic products.

249
Q

Given that NPY and GnRH are large hydrophilic molecules, their receptors are probably in the:

A) cytosol

B) nucleus

C) mitochondria

D) cell membrane

A

[D] NPY and GnRH are large hydrophilic molecules. Small hydrophobic signaling molecules can cross the cell membrane to bind receptors in the cytoplasm; large hydrophilic molecules cannot.

250
Q

Pericytes, smooth muscle cells, and fibroblasts were growth-arrested—that is, treated so that they would not divide but other metabolic processes would functional normally. The pericytes uses in these experiments were probably in which phase of the cell cycle?

A) Telophase

B) Metaphase

C) Anaphase

D) Interphase

A

[D] The cells would not divide, but they would still functional normally. Options A, B, and C are phases of mitosis and occur during cell division. Interphase is a phase in the cell cycle between cell divisions and is the phase in which the cells obtains nutrients, grows, reads its DNA, and conducts other normal cell functions.